Incorrect Questions 2 Flashcards

1
Q

To which of the following matters would materiality limits not apply when obtaining written management representations?
A. Losses from sales commitments.
B. Unasserted claims and assessments.
C. Fraud involving management.
D. Noncompliance with contractual agreements.

A

C. Fraud involving management.
Materiality limits would not apply when obtaining written management representations involving management fraud. Because of their nature and related implications for the control environment, any fraud committed by management is considered to be extremely serious and no materiality limits would apply.

How well did you know this?
1
Not at all
2
3
4
5
Perfectly
2
Q

Which of the following statements would most likely be included among the written client representations obtained by the auditor?
A. Compensating balances and other arrangements involving restrictions on cash balances have been disclosed.
B. Management acknowledges responsibility for illegal actions committed by employees.
C. Sufficient evidential matter has been made available to permit the issuance of an unqualified opinion.
D. Management acknowledges that there are no material weaknesses in the internal control.

A

A. Compensating balances and other arrangements involving restrictions on cash balances have been disclosed.

The management representations letter would appropriately include statements about the disclosure of compensating balances and other arrangements involving restrictions on cash balances.

How well did you know this?
1
Not at all
2
3
4
5
Perfectly
3
Q

In obtaining written representations from management, materiality limits ordinarily would apply to representations related to

A.  Amounts concerning related party transactions.
B.  Fraud involving members of management.
C.  The availability of financial records.
D.  The completeness of minutes of directors' meetings.
A

A. Amounts concerning related party transactions.

The reporting requirements applicable to related party issues implicitly involve materiality considerations in evaluating the fairness of the financial statements.

How well did you know this?
1
Not at all
2
3
4
5
Perfectly
4
Q

Which of the following matters would an auditor most likely include in a management representations letter?
A. Communications with the audit committee concerning weaknesses in internal control structure.
The auditor is required to communicate directly with the audit committee (or others charged with governance). Management would not be required to include such communications in the management representation letter.
B. The completeness and availability of minutes of stockholders’ and directors’ meetings.
C. Plans to acquire or merge with other entities in the subsequent year.
D. Management’s acknowledgment of its responsibility for the detection of all employee fraud.

A

B. The completeness and availability of minutes of stockholders’ and directors’ meetings.

How well did you know this?
1
Not at all
2
3
4
5
Perfectly
5
Q

Key Co. plans to present comparative financial statements for the years ended December 31, 2005, and 2006, respectively. Smith, CPA, audited Key’s financial statements for both years and plans to report on the comparative financial statements on May 1, 2007. Key’s current management team was not present until January 1, 2006. What period of time should be covered by Key’s management representation letter?
A. January l, 2005, through December 31, 2006.
B. January 1, 2005, through May 1, 2007.
C. January 1, 2006, through December 31, 2006.
D. January 1, 2006, through May 1, 2007.

A

B. January 1, 2005, through May 1, 2007.

The management representation letter should address all periods covered by the auditor’s report.

How well did you know this?
1
Not at all
2
3
4
5
Perfectly
6
Q
Wilson, CPA, completed the field work of the audit of Abco's December 31, 2009, financial statements on March 6, 2010, at which time Wilson believed that sufficient appropriate audit evidence had been obtained to support the auditor's opinion. However, a subsequent event requiring adjustment to the 2009 financial statements occurred on April 10, 2010, and came to Wilson's attention on April 24, 2010, which preceded the issuance of the audit report on Abco's 2009 financial statements. If the adjustment is made without disclosure of the event, Wilson's report ordinarily should be dated
	A.  March 6, 2010.
	B.  April 10, 2010.
	C.  April 24, 2010.
	D.  Using dual dating.
A

A. March 6, 2010.

The Professional Standards indicate that, “The auditor’s report should not be dated earlier than the date on which the auditor has obtained sufficient appropriate audit evidence to support the opinion. When a subsequent event occurs requiring adjustment of the financial statements but no disclosure is made, the report will still be dated when sufficient appropriate audit evidence had been obtained, that is, March 6, 2010.

How well did you know this?
1
Not at all
2
3
4
5
Perfectly
7
Q

Zero Corp. suffered a loss that would have a material effect on its financial statements on an uncollectible trade account receivable due to a customer’s bankruptcy.
This occurred suddenly, due to a natural disaster ten days after Zero’s balance sheet date, but one month before the issuance of the financial statements and the auditor’s report. Under these circumstances,

  The financial statements should be adjusted  	  The event requires financial statement disclosure, but no adjustment  	  The auditor's report should be modified for a lack of consistency  
	 Yes 	 No 	 No 
	 Yes 	 No 	 Yes 
	 No 	 Yes 	 Yes 
	 No 	 Yes 	 No
A

The financial statements should be adjusted No

The event requires financial statement disclosure, but no adjustment Yes

The auditor’s report should be modified for a lack of consistency No

How well did you know this?
1
Not at all
2
3
4
5
Perfectly
8
Q

Subsequent to issuing a report on audited financial statements, a CPA discovers that the accounts receivable confirmation process omitted a number of accounts that are material, in the aggregate. Which of the following actions should the CPA take immediately?
A. Bring the matter to the attention of the board of directors or audit committee.
B. Withdraw the auditor’s report from those persons currently relying on it.
C. Perform alternative procedures to verify account balances.
D. Discuss the potential financial statement adjustments with client management.

A

C. Perform alternative procedures to verify account balances.

How well did you know this?
1
Not at all
2
3
4
5
Perfectly
9
Q

When using confirmations to provide evidence about the completeness assertion for accounts payable, the appropriate population most likely would be
A. Vendors with whom the entity has previously done business.
B. Amounts recorded in the accounts payable subsidiary ledger.
C. Payees of checks drawn in the month after the year end.
D. Invoices filed in the entity’s open invoice file.

A

When using confirmations to provide evidence about the completeness assertion for accounts payable, the appropriate population most likely would be
A. Vendors with whom the entity has previously done business.

How well did you know this?
1
Not at all
2
3
4
5
Perfectly
10
Q

Which of the following procedures would an auditor least likely perform before the balance sheet date?
A. Confirmation of accounts payable.
B. Observation of merchandise inventory.
C. Assessment of control risk.
D. Identification of related parties.

A

A. Confirmation of accounts payable.

How well did you know this?
1
Not at all
2
3
4
5
Perfectly
11
Q

Which of the following procedures would an auditor most likely perform in searching for unrecorded liabilities?
A. Trace a sample of accounts payable entries recorded just before year end to the unmatched receiving report file.
B. Compare a sample of purchase orders issued just after year end with the year-end accounts payable trial balance.
C. Vouch a sample of cash disbursements recorded just after year end to receiving reports and vendor invoices.
D. Scan the cash disbursements entries recorded just before year end for indications of unusual transactions.

A

C. Vouch a sample of cash disbursements recorded just after year end to receiving reports and vendor invoices.

Vouching a sample of cash disbursements recorded just after year end to receiving reports and vendor invoices would enable the auditor to determine if the goods were actually received or owned before year end. As a result, the amounts paid after year end would need to be accrued as year-end liabilities.

How well did you know this?
1
Not at all
2
3
4
5
Perfectly
12
Q
To determine whether accounts payable are complete, an auditor performs a test to verify that all merchandise received is recorded. The population of documents for this test consists of all
	A.  Vendor's invoices.
	B.  Purchase orders.
	C.  Receiving reports.
	D.  Canceled checks.
A

C. Receiving reports.

To verify that all merchandise received is recorded, you have to start with the population of merchandise receipts, or receiving reports. You can then trace each receiving report to a recorded vendor invoice.

How well did you know this?
1
Not at all
2
3
4
5
Perfectly
13
Q

While performing a test of details during an audit, an auditor determined that the sample results supported the conclusion that the recorded account balance was materially misstated. It was, in fact, not materially misstated.
This situation illustrates the risk of

A.  Assessing control risk too high.
B.  Assessing control risk too low.
C.  Incorrect rejection.
D.  Incorrect acceptance.
A

C. Incorrect rejection.

Deciding that the sample results support the conclusion that the balance is materially misstated when it is not is an illustration of the risk of incorrect rejection. As a result, the auditor will perform additional and unnecessary work.

How well did you know this?
1
Not at all
2
3
4
5
Perfectly
14
Q

The risk of incorrect acceptance and the likelihood of assessing control risk too low relate to the
A. Effectiveness of the audit.
B. Efficiency of the audit.
C. Preliminary estimates of materiality levels.
D. Allowable risk of tolerable error.

A

A. Effectiveness of the audit.

The risk of incorrect acceptance and the likelihood of assessing control risk too low both relate to the effectiveness of the audit.

How well did you know this?
1
Not at all
2
3
4
5
Perfectly
15
Q

When performing a substantive test of a random sample of cash disbursements, an auditor is supplied with a photocopy of vendor invoices supporting the disbursements for one particular vendor, rather than the original invoices. The auditor is told that the vendor’s original invoices have been misplaced. What should the auditor do in response to this situation?
A. Randomly increase the number of items in the substantive test to increase the reliance that may be placed on the overall test.
B. Reevaluate the risk of fraud and design alternate tests for the related transactions.
C. Increase testing by agreeing more of the payments to this particular vendor to the photocopies of its invoices.
D. Count the missing original documents as misstatements and project the total amount of the error based on the size of the population and the dollar amount of the errors.

A

B. Reevaluate the risk of fraud and design alternate tests for the related transactions.

How well did you know this?
1
Not at all
2
3
4
5
Perfectly
16
Q

Which of the following statements about audit sampling risks is correct for a nonissuer?
A. Nonsampling risk arises from the possibility that, when a substantive test is restricted to a sample, conclusions might be different than if the auditor had tested each item in the population.
B. Nonsampling risk can arise because an auditor failed to recognize misstatements.
C. Sampling risk is derived from the uncertainty in applying audit procedures to specific risks.
D. Sampling risk includes the possibility of selecting audit procedures that are not appropriate to achieve the specific objective.

A

B. Nonsampling risk can arise because an auditor failed to recognize misstatements.

How well did you know this?
1
Not at all
2
3
4
5
Perfectly
17
Q

An auditor may decide to increase the risk of incorrect rejection when
A. Increased reliability from the sample is desired.
B. Many differences (audit value minus recorded value) are expected.
C. Initial sample results do not support the planned level of control risk.
D. The cost and effort of selecting additional sample items are low.

A

D. The cost and effort of selecting additional sample items are low.

How well did you know this?
1
Not at all
2
3
4
5
Perfectly
18
Q

An advantage that using statistical sampling has over nonstatistical sampling is that statistical sampling helps an auditor to
A. Minimize the failure to detect errors and irregularities.
B. Eliminate the risk of nonsampling errors.
C. Reduce the level of audit risk and materiality to a relatively low amount.
D. Measure the sufficiency of the evidential matter obtained.

A

D. Measure the sufficiency of the evidential matter obtained.
Statistical sampling allows an auditor to:
1) design an efficient sample;
2) measure the sufficiency of the evidential matter obtained; and
3) evaluate the sample results.

How well did you know this?
1
Not at all
2
3
4
5
Perfectly
19
Q

An advantage that using statistical sampling methods have over nonstatistical sampling methods in tests of controls is that the statistical methods
A. Can more easily convert the sample into a dual-purpose test useful for substantive testing.
B. Eliminate the need to use judgment in determining appropriate sample sizes.
C. Afford greater assurance than a nonstatistical sample of equal size.
D. Provide an objective basis for quantitatively evaluating sample risk.

A

D. Provide an objective basis for quantitatively evaluating sample risk.

How well did you know this?
1
Not at all
2
3
4
5
Perfectly
20
Q

Hill has decided to use (PPS) sampling, sometimes called dollar-unit sampling, in the audit of a client’s accounts receivable balances. Hill plans to use the following PPS sampling table:
Reliability Factors for Errors of Overstatement
Risk of Incorrect Acceptance
Number of Overstatements 1% 5% 10% 15% 20%
0 4.61 3.00 2.31 1.90 1.61
1 6.64 4.75 3.89 3.38 3.00
2 8.41 6.30 5.33 4.72 4.28
3 10.05 7.76 6.69 6.02 5.52
4 11.61 9.16 8.00 7.27 6.73

ADDITIONAL INFORMATION

Tolerable misstatement (net of the
effect of expected misstatements) $ 24,000
Risk of incorrect acceptance 20%
Number of misstatements allowed 1
Recorded amount of accounts receivable $240,000
Number of accounts 360

What sample size should Hill use?

A. 120
B. 108
C. 60
D. 30

A

D. 30
n = (Reliability factor (from tables) × Book value)/Tolerable misstatement, net of expected misstatements

(3 × $240,000) / $24,000 = 30

How well did you know this?
1
Not at all
2
3
4
5
Perfectly
21
Q

In a PPS sampling application, the sampling interval was $6,000. The auditor discovered that a selected account receivable having a recorded amount of $5,000 had an audit amount of $1,000. What was the projected error associated with this sample?

A. $ 4,000
B. $ 1,200
C. $ 4,800
D. $ 3,200

A

C. $ 4,800
projected error = sampling interval * tainting % = 6,000 * 80%.

tainting % = (recorded amount - audit amount)/recorded amount = (5,000 - 1,000)/5,000 = 80%.

How well did you know this?
1
Not at all
2
3
4
5
Perfectly
22
Q

Which of the following is the primary objective of probability proportional to sample size?

A. To identify overstatement errors.
B. To increase the proportion of smaller-value items in the sample.
C. To identify items where controls were not properly applied.
D. To identify zero and negative balances.

A

A. To identify overstatement errors.

How well did you know this?
1
Not at all
2
3
4
5
Perfectly
23
Q

An auditor discovered that a client’s accounts receivable turnover is substantially lower for the current year than for the prior year.
This may indicate that

A.  Fictitious credit sales have been recorded during the year.
B.  Employees have stolen inventory just before the year end.
C.  The client recently tightened its credit-granting policies.
D.  An employee has been lapping receivables in both years.
A

A. Fictitious credit sales have been recorded during the year.

How well did you know this?
1
Not at all
2
3
4
5
Perfectly
24
Q

When an auditor decides to confirm accounts receivable balances rather than individual invoices, it most likely would be beneficial to include with the confirmations

A.  Copies of the client's shipping documents that support the account balances.
B.  Lists of the customers' recent payments that the client has already recorded.
C.  Client-prepared statements of account that show the details of the account balances.
D.  Copies of the customers' purchase orders that support the account balances.
A

C. Client-prepared statements of account that show the details of the account balances.

How well did you know this?
1
Not at all
2
3
4
5
Perfectly
25
Q

An auditor’s tests of controls for completeness for the revenue cycle usually include determining whether

A.  Each receivable is collected subsequent to the year end.
B.  An invoice is prepared for each shipping document.
C.  Each invoice is supported by a customer purchase order.
D.  Each credit memo is properly approved.
A

B. An invoice is prepared for each shipping document.

How well did you know this?
1
Not at all
2
3
4
5
Perfectly
26
Q

In auditing accounts receivable, the negative form of confirmation request most likely would be used when
A. The total recorded amount of accounts receivable is immaterial to the financial statements taken as a whole.
B. Response rates in prior years to properly designed positive confirmation requests were inadequate.
C. Recipients are unlikely to give the confirmation requests proper consideration.
D. The combined assessed level of inherent risk and control risk relative to accounts receivable is low.

A

D. The combined assessed level of inherent risk and control risk relative to accounts receivable is low.

How well did you know this?
1
Not at all
2
3
4
5
Perfectly
27
Q

Which of the following procedures would an auditor most likely perform for year-end accounts receivable confirmations when the auditor did not receive replies to second requests?
A. Review the cash receipts journal for the month prior to the year end.
B. Intensify the study of internal control structure concerning the revenue cycle.
C. Increase the assessed level of detection risk for the existence assertion.
D. Inspect the shipping records documenting the merchandise sold to the debtors.

A

D. Inspect the shipping records documenting the merchandise sold to the debtors.

How well did you know this?
1
Not at all
2
3
4
5
Perfectly
28
Q

When an auditor does not receive replies to positive requests for year-end accounts receivable confirmations, the auditor most likely would
A. Inspect the allowance account to verify whether the accounts were subsequently written off.
B. Increase the assessed level of detection risk for the valuation and completeness assertions.
C. Ask the client to contact the customers to request that the confirmations be returned.
D. Increase the assessed level of inherent risk for the revenue cycle.

A

C. Ask the client to contact the customers to request that the confirmations be returned.

How well did you know this?
1
Not at all
2
3
4
5
Perfectly
29
Q

Which of the following most likely would give the most assurance concerning the valuation assertion of accounts receivable?
A. Tracing amounts in the subsidiary ledger to details on shipping documents.
B. Comparing receivable turnover ratios to industry statistics for reasonableness.
C. Inquiring about receivables pledged under loan agreements.
D. Assessing the allowance for uncollectible accounts for reasonableness.

A

D. Assessing the allowance for uncollectible accounts for reasonableness.

How well did you know this?
1
Not at all
2
3
4
5
Perfectly
30
Q

Which of the following circumstances most likely would cause an auditor to believe that material misstatements may exist in an entity’s financial statements?
A. Accounts receivable confirmation requests yield significantly fewer responses than expected.
B. Audit trails of computer-generated transactions exist only for a short time.
C. The chief financial officer does not sign the management representation letter until the after the field work is completed.
D. Management consults with other accountants about significant accounting matters.

A

A. Accounts receivable confirmation requests yield significantly fewer responses than expected.

How well did you know this?
1
Not at all
2
3
4
5
Perfectly
31
Q
In confirming a client's accounts receivable in prior years, an auditor discovered many differences between recorded account balances and confirmation replies. These differences were resolved and were not misstatements. In defining the sampling unit for the current year's audit, the auditor most likely would choose
	A.  Customers with credit balances.
	B.  Small account balances.
	C.  Individual overdue balances.
	D.  Individual invoices.
A

D. Individual invoices.

How well did you know this?
1
Not at all
2
3
4
5
Perfectly
32
Q

An auditor desired to test credit approval on 10,000 sales invoices processed during the year. The auditor designed a statistical sample that would provide 1% risk of assessing control risk too low (99% confidence) that not more than 7% of the sales invoices lacked approval.
The auditor estimated from previous experience that about 2.5% of the sales invoices lacked approval. A sample of 200 invoices was examined and 7 of them were lacking approval.
The auditor then determined the achieved upper precision limit to be 8%.

The allowance for sampling risk was

A.  5.5%.
B.  4.5%.
C.  3.5%.
D.  1%.
A

B. 4.5%.

The allowance for sampling risk = achieved upper precision limit (8%)-less sample error rate (3.5%) or 4.5%.

How well did you know this?
1
Not at all
2
3
4
5
Perfectly
33
Q

The sample size of a test of controls varies inversely with
Expected population deviation rate Tolerable rate
Yes Yes
No No
Yes No
No Yes

A

Expected population deviation rate: No Tolerable rate: Yes

How well did you know this?
1
Not at all
2
3
4
5
Perfectly
34
Q

Which of the following statements is correct concerning statistical sampling in tests of controls?
A. As the population size increases, the sample size should increase proportionately.
B. Deviations from specific internal control procedures at a given rate ordinarily result in misstatements at a lower rate.
C. There is an inverse relationship between the expected population deviation rate and the sample size.
D. In determining tolerable rate, an auditor considers detection risk and the sample size.

A

B. Deviations from specific internal control procedures at a given rate ordinarily result in misstatements at a lower rate.

How well did you know this?
1
Not at all
2
3
4
5
Perfectly
35
Q

An auditor who uses statistical sampling for attributes in testing internal controls should reduce the planned reliance on a prescribed control when the
A. Sample rate of deviation plus the allowance for sampling risk equals the tolerable rate.
B. Sample rate of deviation is less than the expected rate of deviation used in planning the sample.
C. Tolerable rate less the allowance for sampling risk exceeds the sample rate of deviation.
D. Sample rate of deviation plus the allowance for sampling risk exceeds the tolerable rate.

A

D. Sample rate of deviation plus the allowance for sampling risk exceeds the tolerable rate.

How well did you know this?
1
Not at all
2
3
4
5
Perfectly
36
Q

In determining the number of documents to select for a test to obtain assurance that all sales returns have been properly authorized, an auditor should consider the tolerable rate of deviation from the control activity.
The auditor should also consider the
I. Likely rate of deviations.

II. Allowable risk of assessing control risk too high.

A.  I only.
B.  II only.
C.  Both I and II.
D.  Either I or II.
A

A. I only.

How well did you know this?
1
Not at all
2
3
4
5
Perfectly
37
Q

For which of the following audit tests would a CPA most likely use attribute sampling?
A. Identifying entries posted to incorrect accounts.
B. Estimating the amount in an expense account.
C. Evaluating the reasonableness of depreciation expense.
D. Selecting receivables for confirmation of account balances.

A

A. Identifying entries posted to incorrect accounts.

Attributes sampling is used to estimate the proportion of a characteristic in a population. As a result, it is typically used for tests of controls and substantive tests of transactions. Identifying entries posted to incorrect accounts is a substantive test of transaction in which attribute sampling could be utilized.

How well did you know this?
1
Not at all
2
3
4
5
Perfectly
38
Q
To determine the sample size for a test of controls, an auditor should consider the tolerable deviation rate, the allowable risk of assessing control risk too low, and the
	A.  Expected deviation rate.
	B.  Upper precision limit.
	C.  Risk of incorrect acceptance.
	D.  Risk of incorrect rejection.
A

A. Expected deviation rate.

How well did you know this?
1
Not at all
2
3
4
5
Perfectly
39
Q

An auditor desired to test credit approval on 10,000 sales invoices processed during the year. The auditor designed a statistical sample that would provide 1% risk of assessing control risk too low (99% confidence) that not more than 7% of the sales invoices lacked approval.
The auditor estimated from previous experience that about 2.5% of the sales invoices lacked approval. A sample of 200 invoices was examined and 7 of them were lacking approval.
The auditor then determined the achieved upper precision limit to be 8%.
In the evaluation of this sample, the auditor decided to increase the level of the preliminary assessment of control risk because the

A.  Tolerable rate (7%) was less than the achieved upper precision limit (8%).
B.  Expected deviation rate (7%) was more than the percentage of errors in the sample (3.5%).
C.  Achieved upper precision limit (8%) was more than the percentage of errors in the sample (3.5%).
D.  Expected deviation rate (2.5%) was less than the tolerable rate (7%).
A

A. Tolerable rate (7%) was less than the achieved upper precision limit (8%).

In a test of controls, the auditor compares the tolerable rate (the maximum rate of deviations that the auditor would be willing to accept without altering the planned assessment of control risk) to the achieved upper precision limit (the maximum deviation rate per the sample).

If the achieved upper precision limit is greater than the tolerable rate, the control cannot be relied upon and the control risk assessment will be increased.

How well did you know this?
1
Not at all
2
3
4
5
Perfectly
40
Q

In statistical sampling methods used in substantive testing, an auditor most likely would stratify a population into meaningful groups if
A. Probability proportional to size (PPS) sampling is used.
B. The population has highly variable recorded amounts.
C. The auditor’s estimated tolerable misstatement is extremely small.
D. The standard deviation of recorded amounts is relatively small.

A

B. The population has highly variable recorded amounts.

How well did you know this?
1
Not at all
2
3
4
5
Perfectly
41
Q

When using classical variables sampling for estimation, an auditor normally evaluates the sampling results by calculating the possible error in either direction.
This statistical concept is known as

A.  Precision.
B.  Reliability.
C.  Projected error.
D.  Standard deviation.
A

A. Precision.

How well did you know this?
1
Not at all
2
3
4
5
Perfectly
42
Q

The group engagement partner decides not to refer to the audit of a component auditor who audited a subsidiary of the group auditor’s client. After making inquiries about the component auditor’s professional reputation and independence, the group engagement partner most likely would
A. Add an emphasis-of-matter paragraph to the auditor’s report indicating that the subsidiary’s financial statements are not material to the consolidated financial statements.
B. In the engagement letter that the group auditor assumes no responsibility for the component auditor’s work and opinion.
C. Obtain written permission from the component auditor to omit the reference in the group engagement partner’s audit report.
D. Contact the component auditor and review the audit programs and documentation pertaining to the subsidiary.

A

D. Contact the component auditor and review the audit programs and documentation pertaining to the subsidiary.

How well did you know this?
1
Not at all
2
3
4
5
Perfectly
43
Q

Which of the following procedures would the group auditor most likely perform after deciding to make reference to a component auditor who audited a subsidiary of the entity?
A. Review the audit documentation and the audit programs of the component auditor.
A review of the audit documentation and audit programs of the component auditor would be more likely to be performed when the group engagement partner does NOT plan to make reference to the component auditor.
B. Visit the component auditor and discuss the results of the other CPA’s audit procedures.
C. Make inquiries about the professional reputation and independence of the component auditor.
D. Determine that the component auditor has a sufficient understanding of the subsidiary’s internal control.

A

C. Make inquiries about the professional reputation and independence of the component auditor.

How well did you know this?
1
Not at all
2
3
4
5
Perfectly
44
Q

In which of the following situations would a group engagement partner least likely make reference to a component auditor who audited a subsidiary of the entity?
A. The component auditor was retained by the group auditor and the work was performed under the group auditor’s guidance and control.
B. The group auditor finds it impracticable to review the component auditor’s work or otherwise be satisfied as to the component auditor’s work.
C. The group engagement partner is unable to be satisfied as to the independence and professional reputation of the component auditor.
D. The principal auditor is unable to be satisfied as to the independence and professional reputation of the other auditor.

A

A. The component auditor was retained by the group auditor and the work was performed under the group auditor’s guidance and control.

Group engagement partner is not as likely to refer to a component auditor when the component auditor was retained by the group auditor and the work was performed under the group auditor’s guidance and control.

How well did you know this?
1
Not at all
2
3
4
5
Perfectly
45
Q

In the group auditor’s report, the group engagement partner decides not to make reference to a component auditor who audited a client’s subsidiary. The group auditor could justify this decision if, among other requirements, the group engagement partner
A. Issues an unmodified opinion on the consolidated financial statements.
B. Learns that the component auditor issued an unmodified opinion on the subsidiary’s financial statements.
C. Is unable to review the audit programs and audit documentation of the component auditor.
D. Is satisfied as to the independence and professional reputation of the component auditor.

A

D. Is satisfied as to the independence and professional reputation of the component auditor.

How well did you know this?
1
Not at all
2
3
4
5
Perfectly
46
Q

When unaudited financial statements of a nonpublic entity are presented in comparative form with audited financial statements in the subsequent year, the unaudited financial statements should be clearly marked to indicate their status and

I. The report on the unaudited financial statements should be reissued.
II. The report on the audited financial statements should include a separate paragraph describing the responsibility assumed for the unaudited financial statements.

A.  I only.
B.  II only.
C.  Both I and II.
D.  Either I or II.
A

D. Either I or II.

How well did you know this?
1
Not at all
2
3
4
5
Perfectly
47
Q

King, CPA, was engaged to audit the financial statements of Newton Company after its fiscal year had ended. King neither observed the inventory count nor confirmed the receivables by direct communication with debtors, but was satisfied concerning both after applying alternative procedures.
King’s auditor’s report most likely contained a(n)

A.  Qualified opinion
B.  Disclaimer of opinion.
C.  Unmodified opinion.
D.  Unmodified opinion with an other-matter paragraph.
A

C. Unmodified opinion.

As long as the auditor is satisfied regarding the fair presentation of the accounts and financial statements in accordance with GAAP, an unmodified opinion may be expressed.

How well did you know this?
1
Not at all
2
3
4
5
Perfectly
48
Q

Digit Co. uses the FIFO method of costing for its international subsidiary’s inventory and LIFO for its domestic inventory. Under these circumstances, the auditor’s report on Digit’s financial statements should express an
A. Unmodified opinion.
B. Opinion qualified because of a lack of consistency.
C. Opinion qualified because of a departure from GAAP.
D. Adverse opinion.

A

A. Unmodified opinion.

Use of more than one costing method for different inventories is not prohibited by GAAP. As a result, there is no GAAP departure and thus, no reason to qualify the auditor’s report.

How well did you know this?
1
Not at all
2
3
4
5
Perfectly
49
Q

Eagle Company’s financial statements contain a departure from generally accepted accounting principles because, due to unusual circumstances, the statements would otherwise be misleading.
The auditor should express an opinion that is

A.  Unmodified, but should not mention the departure in the auditor's report.
B.  Unmodified, and should describe the departure in a separate paragraph.
C.  Qualified, and should describe the departure in a separate paragraph.
D.  Qualified or adverse, depending on materiality, and should describe the departure in a separate paragraph.
A

B. Unmodified, and should describe the departure in a separate paragraph.

How well did you know this?
1
Not at all
2
3
4
5
Perfectly
50
Q

For an entity’s financial statements to be presented fairly in conformity with generally accepted accounting principles, the principles selected should
A. Be applied on a basis consistent with those followed in the prior year.
B. Be approved by the Auditing Standards Board or the appropriate industry subcommittee.
C. Reflect transactions in a manner that presents the financial statements within a range of acceptable limits.
D. Match the principles used by most other entities within the entity’s particular industry.

A

C. Reflect transactions in a manner that presents the financial statements within a range of acceptable limits.

How well did you know this?
1
Not at all
2
3
4
5
Perfectly
51
Q

An auditor may not issue a qualified opinion when
A. An accounting principle at variance with GAAP is used.
B. The auditor lacks independence with respect to the audited entity.
C. A scope limitation prevents the auditor from completing an important audit procedure.
D. The auditor’s report refers to the work of a specialist.

A

B. The auditor lacks independence with respect to the audited entity.
The professional standards require that the auditor be independent. When the auditor is not independent, the auditor is precluded from issuing any type of report other than a disclaimer.

How well did you know this?
1
Not at all
2
3
4
5
Perfectly
52
Q

Which of the following phrases should be included in the opinion paragraph when an auditor expresses a qualified opinion?
When read in conjunction with Note X With the foregoing explanation
Yes No
No Yes
Yes Yes
No No

A

No No
The opinion paragraph of a qualified opinion should include the phrase “except for . . .” It should not include either of the two phrases listed.

How well did you know this?
1
Not at all
2
3
4
5
Perfectly
53
Q

An auditor most likely would modify the audit report if the entity’s financial statements include a footnote on related party transactions
A. Disclosing loans to related parties at interest rates significantly below prevailing market rates.
B. Describing an exchange of real estate for similar property in a non-monetary related party transaction.
C. Stating that a particular related party transaction occurred on terms equivalent to those that would have prevailed in an arm’s-length transaction.
D. Presenting the dollar volume of related party transactions and the effects of any change in the method of establishing terms from prior periods.

A

C. Stating that a particular related party transaction occurred on terms equivalent to those that would have prevailed in an arm’s-length transaction.

In general, it is not possible to determine whether or not such transactions were conducted on terms equivalent to those in an arm’s-length transaction.

How well did you know this?
1
Not at all
2
3
4
5
Perfectly
54
Q

A limitation on the scope of an audit sufficient to preclude an unmodified opinion will usually result when management
A. Is unable to obtain audited financial statements supporting the entity’s investment in a foreign subsidiary.
B. Refuses to disclose in the notes to the financial statements related party transactions authorized by the board of directors.
C. Does not sign an engagement letter specifying the responsibilities of both the entity and the auditor.
D. Fails to correct a significant deficiency communicated to the audit committee after the prior year’s audit.

A

A. Is unable to obtain audited financial statements supporting the entity’s investment in a foreign subsidiary.

How well did you know this?
1
Not at all
2
3
4
5
Perfectly
55
Q

In which of the following circumstances would an auditor not express an unmodified opinion?
A. There has been a material change in accounting principles between periods.
B. Quarterly financial data required by the SEC has been omitted.
C. The auditor wishes to emphasize an unusually important subsequent event.
D. The auditor is unable to obtain audited financial statements of a consolidated investee.

A

D. The auditor is unable to obtain audited financial statements of a consolidated investee.

An auditor may not express an unmodified opinion if the auditor is unable to obtain audited financial statements of a consolidated investee. This represents a scope limitation, which would warrant either a qualified opinion or a disclaimer.

How well did you know this?
1
Not at all
2
3
4
5
Perfectly
56
Q

Zag Co. issues financial statements that present financial position and results of operations but Zag omits the related statement of cash flows. Zag would like to engage Brown, CPA, to audit its financial statements without the statement of cash flows although Brown’s access to all of the information underlying the basic financial statements will not be limited. Under these circumstances, Brown most likely would
A. Add an emphasis-of-matter paragraph to the standard auditor’s report that justifies the reason for the omission.
B. Refuse to accept the engagement as proposed because of the client-imposed scope limitation.
C. Explain to Zag that the omission requires a qualification of the auditor’s opinion.
D. Prepare the statement of cash flows as an accommodation to Zag and express an unmodified opinion.

A

C. Explain to Zag that the omission requires a qualification of the auditor’s opinion.

When an entity omits a statement of cash flows, the auditor may accept an engagement to audit the other financial statements, but should qualify the opinion, since a statement of cash flows is required when general-purpose financial statements present financial position and results of operation.

How well did you know this?
1
Not at all
2
3
4
5
Perfectly
57
Q

Tread Corp. accounts for the effect of a material accounting change prospectively when the inclusion of the cumulative effect of the change is required in the current year.
The auditor would choose between expressing a(n)

A.  Qualified opinion or a disclaimer of opinion.
B.  Disclaimer of opinion or an unmodified opinion with an emphasis-of-paragraph.
C.  Unmodified opinion with an emphasis-of-matter paragraph and an adverse opinion.
D.  Adverse opinion and a qualified opinion.
A

D. Adverse opinion and a qualified opinion.

How well did you know this?
1
Not at all
2
3
4
5
Perfectly
58
Q

When an auditor expresses an adverse opinion, the opinion paragraph should include
A. The principal effects of the departure from generally accepted accounting principles.
B. A direct reference to a separate paragraph disclosing the basis for the opinion.
C. The substantive reasons that the financial statements are misleading.
D. A description of the uncertainty or scope limitation that prevents an unmodified opinion.

A

B. A direct reference to a separate paragraph disclosing the basis for the opinion.

How well did you know this?
1
Not at all
2
3
4
5
Perfectly
59
Q

An auditor would express an unmodified opinion with an emphasis-of-matter paragraph added to the auditor’s report for
An unjustified accounting change A material weakness in the internal control structure
Yes Yes
Yes No
No Yes
No No

A

No No
An unjustified change in accounting principles is a GAAP departure that would result in a qualified or adverse opinion. A material weakness in internal control should be reported to those charged with governance, but would not be reported in an unmodified audit report.

How well did you know this?
1
Not at all
2
3
4
5
Perfectly
60
Q

Which of the following would a successor auditor ask the predecessor auditor to provide after accepting an audit engagement?
A. Disagreements between the predecessor auditor and management as to significant accounting policies and principles.
B. The predecessor auditor’s understanding of the reasons for the change of auditors.
C. Facts known to the predecessor auditor that might bear on the integrity of management.
D. Matters that may facilitate the evaluation of financial reporting consistency between the current and prior years.

A

D. Matters that may facilitate the evaluation of financial reporting consistency between the current and prior years.

How well did you know this?
1
Not at all
2
3
4
5
Perfectly
61
Q

Park, CPA, was engaged to audit the financial statements of Tech Co., a new client, for the year ended December 31, 20x1. Park obtained sufficient audit evidence for all of Tech’s financial statement items except Tech’s opening inventory. Due to inadequate financial records, Park could not verify Tech’s January 1, 20x1, inventory balances.
Park’s opinion on Tech’s 20x1 financial statements most likely will be on the

  Balance sheet  	  Income statement  
	 Disclaimer 	 Disclaimer 
	 Unmodified 	 Disclaimer 
	 Disclaimer 	 Adverse 
	 Unmodified 	 Adverse
A

Balance sheet: Unmodified Income statement: Disclaimer

The inability to verify the beginning inventory makes the auditor unable to express an opinion on any financial statement in which inventory is a material component. Beginning inventory is material to cost of goods sold and net income. As a result, the auditor is unable to express an opinion and must disclaim on the income and retained earnings statements and the statement of cash flows. The auditor will, however, be able to render an unmodified opinion on the balance sheet.

How well did you know this?
1
Not at all
2
3
4
5
Perfectly
62
Q

When audited financial statements are presented in a client’s document containing other information, the auditor should
A. Perform inquiry and analytical procedures to ascertain whether the other information is reasonable.
B. Add an emphasis-of-matter paragraph to the auditor’s report without changing the opinion on the financial statements.
C. Perform the appropriate substantive auditing procedures to corroborate the other information.
D. Read the other information to determine that it is consistent with the audited financial statements.

A

D. Read the other information to determine that it is consistent with the audited financial statements.

How well did you know this?
1
Not at all
2
3
4
5
Perfectly
63
Q

If the auditor concludes that the financial statements do not require revision, but the client refuses to revise or eliminate the material inconsistency, the auditor may

A.  Revise the auditor's report to include a separate other-matter paragraph describing the material inconsistency.
B.  Issue an "except for" qualified opinion after discussing the matter with the client's board of directors.
C.  Consider the matter closed because the other information is not in the audited financial statements.
D.  Disclaim an opinion on the financial statements after explaining the material inconsistency in a separate emphasis-of-matter paragraph.
A

A. Revise the auditor’s report to include a separate other-matter paragraph describing the material inconsistency.

How well did you know this?
1
Not at all
2
3
4
5
Perfectly
64
Q

When disclaiming an opinion due to a client-imposed scope limitation, an auditor should indicate in a separate paragraph why the audit did not comply with generally accepted auditing standards. The auditor should also
Modify the Auditor’s
Responsibility Section Omit the opinion paragraph
No Yes
Yes Yes
No No
Yes No

A

Modify the Auditor’s
Responsibility Section: Yes Omit the opinion paragraph: No

The opinion paragraph remains but it indicates that the scope of work was insufficient to support an opinion.

How well did you know this?
1
Not at all
2
3
4
5
Perfectly
65
Q

When there has been a change in accounting principle that materially affects the comparability of the comparative financial statements presented and the auditor concurs with the change, the auditor should
Concur explicitly with the change Issue an “except for” qualified opinion Refer to the change in an emphasis-of-matter paragraph
No No Yes
Yes No Yes
Yes Yes No
No Yes No

A

No No Yes
If a change in accounting principle has occurred and the auditor concurs with the change, the only requirement that must be met is to refer to the change in an emphasis-of-matter paragraph following an otherwise unmodified opinion.
It is not necessary for the auditor to concur explicitly with the change nor is it appropriate for the opinion to be qualified as a result of the change.

How well did you know this?
1
Not at all
2
3
4
5
Perfectly
66
Q

In the first audit of a new client, an auditor was able to extend auditing procedures to gather sufficient evidence about consistency.
Under these circumstances, the auditor should
A. Not report on the client’s income statement.
B. Not refer to consistency in the auditor’s report.
C. State that the consistency standard does not apply.
D. State that the accounting principles have been applied consistently.

A

B. Not refer to consistency in the auditor’s report.

GAAS require that the auditor identify occasions in which the accounting principles have not been applied consistently. No mention is to be made in the report if the consistency requirement is met. If the auditor was able to obtain sufficient evidence about consistency, the auditor’s report should not refer to consistency.

How well did you know this?
1
Not at all
2
3
4
5
Perfectly
67
Q

Investment and property schedules are presented for purposes of additional analysis in an auditor-submitted document. The schedules are not required parts of the basic financial statements, but accompany the financial statements.
When reporting on such supplementary information in relation to the financial statements as a whole, the measurement of materiality is the

A.  Same as that used in forming an opinion on the basic financial statements taken as a whole.
B.  Lesser of the individual schedule of investments or schedule of property, taken by itself.
C.  Greater of the individual schedule of investments or schedule of property, taken by itself.
D.  Combined total of both the individual schedules of investments and property, taken as a whole.
A

A. Same as that used in forming an opinion on the basic financial statements taken as a whole.

How well did you know this?
1
Not at all
2
3
4
5
Perfectly
68
Q

If supplementary information in a document accompanying the basic financial statements has been subjected to auditing procedures, the auditor may include in the auditor’s report on the financial statements an opinion that the accompanying information is fairly stated in
A. Accordance with generally accepted auditing standards.
B. Conformity with generally accepted accounting principles.
C. All material respects in relation to the financial statements as a whole.
D. Accordance with attestation standards expressing a conclusion about management’s assertions.

A

C. All material respects in relation to the financial statements as a whole.

How well did you know this?
1
Not at all
2
3
4
5
Perfectly
69
Q

What is an auditor’s responsibility for supplementary information, which is outside the basic financial statements, but which is required by the FASB?
A. The auditor has no responsibility for required supplementary information, as long as it is outside the basic financial statements.
B. The auditor’s only responsibility for required supplementary information is to determine that such information has not been omitted.
C. The auditor should apply certain limited procedures to the required supplementary information and report deficiencies in, or omissions of, such information.
D. The auditor should apply tests of details of transactions and balances to the required supplementary information and report any material misstatements in such information.

A

C. The auditor should apply certain limited procedures to the required supplementary information and report deficiencies in, or omissions of, such information.

How well did you know this?
1
Not at all
2
3
4
5
Perfectly
70
Q

What is an auditor’s responsibility for supplementary information required by the GASB that is placed outside the basic financial statements?
A. Label the information as unaudited and expand the auditor’s report to include a disclaimer on the information.
B. Add an emphasis-of-matter paragraph and apply limited procedures to the information.
C. Add an other-matter paragraph to the auditor’s report and apply limited procedures to the information.
D. Audit the required supplementary information in accordance with generally accepted governmental auditing standards.

A

C. Add an other-matter paragraph to the auditor’s report and apply limited procedures to the information.

The auditor’s responsibility for supplementary information required by the GASB that is placed outside the basic financial statements is limited to comparing the required supplementary information for consistency with the audited financial statements and reporting on the information in an other-matter paragraph.

How well did you know this?
1
Not at all
2
3
4
5
Perfectly
71
Q

The purpose of an alert to restrict the use of the auditor’s report is best described as
A. Reducing the potential for misunderstanding if the auditor’s report is taken out of the context for which the written communication is intended.
B. Communicating the auditor’s responsibility for enforcing the distribution of the auditor’s written communication after releasing the report.
C. Communicating that the measurement criteria upon which the report is based are understandable only to a limited number of users.
D. Indicating that such an alert is only permitted in certain unusual circumstances.

A

A. Reducing the potential for misunderstanding if the auditor’s report is taken out of the context for which the written communication is intended.

How well did you know this?
1
Not at all
2
3
4
5
Perfectly
72
Q

When adding an alert to restrict the auditor’s report, the auditor should place the alert
A. In the introductory paragraph of the auditor’s report.
B. In the Auditor’s Responsibility section of the auditor’s report.
C. In a paragraph preceding the opinion paragraph.
D. In a paragraph at the end of the auditor’s report.

A

D. In a paragraph at the end of the auditor’s report.

How well did you know this?
1
Not at all
2
3
4
5
Perfectly
73
Q

An alert to restrict the auditor’s report is required when
A. The auditor’s report is modified for a scope limitation that is considered to be material and pervasive.
There is no requirement to restrict the distribution of the auditor’s report for a disclaimer of opinion.
B. The subject matter is based on criteria that are suitable and available to all users.
C. The report is considered to be a by-product to the primary objective of the engagement.
D. The auditor’s report includes an other matter paragraph to clarify the auditor’s responsibilities.

A

C. The report is considered to be a by-product to the primary objective of the engagement.

An alert to restrict the use of the auditor’s report is required when (1) the subject matter is based on criteria that are only suitable or available to a limited number of users; or (2) when the matters are presented in a by-product report that is not the primary objective of the engagement.

How well did you know this?
1
Not at all
2
3
4
5
Perfectly
74
Q

Which of the following statements would not normally be included in a representation letter for a review of interim financial information?

A.  To the best of our knowledge and belief, no events have occurred subsequent to the balance sheet and through the date of this letter that would require adjustment to or disclosure in the interim financial information.
B.  We acknowledge our responsibility for the design and implementation of programs and controls to prevent and detect fraud.
C.  We understand that a review consists principally of performing analytical procedures and making inquiries about the interim financial information.
D.  We have made available to you all financial records and related data.
A

C. We understand that a review consists principally of performing analytical procedures and making inquiries about the interim financial information.

The AICPA’s sample management representation letter for interim financial information does not include a representation about understanding the meaning of a “review” of interim financial information. The nature of such an engagement would be clearly communicated in the required engagement letter, but it would not be a statement of fact by management in response to the auditor’s inquiries.

How well did you know this?
1
Not at all
2
3
4
5
Perfectly
75
Q

Due to a scope limitation, an auditor disclaimed an opinion on the financial statements taken as a whole, but the auditor’s report included a statement that the current asset portion of the entity’s balance sheet was fairly stated.
The inclusion of this statement is

A.  Not appropriate because it may tend to overshadow the auditor's disclaimer of opinion.
B.  Not appropriate because the auditor is prohibited from reporting on only one basic financial statement.
C.  Appropriate provided the auditor's scope paragraph adequately describes the scope limitation.
D.  Appropriate provided the statement is in a separate paragraph preceding the disclaimer of opinion paragraph.
A

A. Not appropriate because it may tend to overshadow the auditor’s disclaimer of opinion.

How well did you know this?
1
Not at all
2
3
4
5
Perfectly
76
Q

When an independent accountant’s report based on a review of interim financial information is presented in a registration statement, a prospectus should include a statement about the accountant’s involvement. This statement should clarify that the
A. Accountant is not an “expert” within the meaning of the Securities Act of 1933.
B. Accountant’s review report is not a “part” of the registration statement within the meaning of the Securities Act of 1933.
C. Accountant performed only limited auditing procedures on the interim financial statements.
A statement about the accountant’s involvement with interim financial information presented in a registration statement should clarify that the accountant’s review report is not a “part” of the registration statement within the meaning of the Securities Act of 1933. The review report already includes a description of the limited auditing procedures performed in a review; it is not necessary to make that disclosure again.
D. Accountant’s review was performed in accordance with standards established by the American Institute of CPAs.

A

B. Accountant’s review report is not a “part” of the registration statement within the meaning of the Securities Act of 1933.

How well did you know this?
1
Not at all
2
3
4
5
Perfectly
77
Q

Green, CPA, is aware that Green’s name is to be included in the interim report of National Company, a publicly held entity. National’s quarterly financial statements are contained in the interim report.
Green has not audited or reviewed these interim financial statements.
Green should request that

I. Green’s name not be included in the communication.
II. The financial statements be marked as unaudited with a notation that no opinion is expressed on them.

A.  I only.
B.  II only.
C.  Both I and II.
D.  Either I or II.
A

D. Either I or II.

How well did you know this?
1
Not at all
2
3
4
5
Perfectly
78
Q

The objective of a review of interim financial information of a public entity is to provide the accountant with a basis for
A. Determining whether the prospective financial information is based on reasonable assumptions.
B. Expressing a limited opinion that the financial information is presented in conformity with the applicable financial reporting framework.
C. Deciding whether to perform substantive audit procedures prior to the balance sheet date.
D. Reporting whether material modifications should be made for such information to conform with the applicable financial reporting framework.

A

D. Reporting whether material modifications should be made for such information to conform with the applicable financial reporting framework.

How well did you know this?
1
Not at all
2
3
4
5
Perfectly
79
Q

When unaudited financial statements are presented in comparative form with audited financial statements in a document filed with the Securities and Exchange Commission (SEC), such statements should be
Marked as “unaudited” Withheld until audited Referred to in the auditor’s report
Yes No No
Yes No Yes
No Yes Yes
No Yes No

A

Marked as “unaudited” Withheld until audited Referred to in the auditor’s report
Yes No No
The presentation of unaudited financial statements with audited financial statements in an SEC filing requires that the unaudited financial statements be marked as “unaudited.” The auditor is not required to withhold the unaudited statements until they are audited or to refer to the unaudited statements in the auditor’s report.

How well did you know this?
1
Not at all
2
3
4
5
Perfectly
80
Q

When planning a review of an audit client’s interim financial statements, which of the following procedures should the accountant perform to update the accountant’s knowledge about the entity’s business and its internal control?

A.  Perform analytical procedures on selected accounts by comparing the interim amounts to the amounts for the previous audited fiscal year end.
B.  Inquire of the entity's outside legal counsel about the status of any previous pending litigation and any new litigation involving the entity.
C.  Select a sample of material revenue transactions occurring during the interim period and examine supporting documentation.
D.  Consider the results of audit procedures performed with respect to the current year's financial statements.
A

D. Consider the results of audit procedures performed with respect to the current year’s financial statements.

How well did you know this?
1
Not at all
2
3
4
5
Perfectly
81
Q

The financial statements of KCP America, a U.S. entity, are prepared for inclusion in the consolidated financial statements of its non-U.S. parent. These financial statements are prepared in conformity with the accounting principles generally accepted in the parent’s country and are for use only in that country.
How may KCP America’s auditor report on these financial statements?

I. A U.S.-style report (without revision).

II. A U.S.-style report revised to reference the accounting principles of the parent’s country.

III. The report form of the parent’s country.

  I  	  II  	  III  
	 Yes 	 No 	 No 
	 No 	 Yes 	 No 
	 Yes 	 No 	 Yes 
	 No 	 Yes 	 Yes
A

No Yes Yes

Provided that the financial statements prepared in conformity with another country’s GAAP are for use only outside the United States, KCP America’s auditor may issue either a U.S.-style report revised to reference the accounting principles (financial reporting framework) of the parent’s country or the report form of the parent’s country.

How well did you know this?
1
Not at all
2
3
4
5
Perfectly
82
Q

An auditor may report on condensed financial statements that are derived from a complete set of audited financial statements only if the auditor
A. Expresses an unqualified opinion on the audited financial statements from which the condensed financial statements are derived.
B. Indicates whether the information is consistent in all material respects in relation to the complete financial statements.
C. Determines that the condensed financial statements include all the disclosures necessary for the complete set of financial statements.
D. Presents the condensed financial statements in comparative form with the prior year’s condensed financial statements.

A

B. Indicates whether the information is consistent in all material respects in relation to the complete financial statements.
Condensed financial statements are not GAAP financial statements but a shortened and summarized version. As a result, the auditor must report on condensed financial statements with different wording. Specifically, the auditor must indicate whether the information is consistent in all material respects in relation to the complete (GAAP) financial statements.

How well did you know this?
1
Not at all
2
3
4
5
Perfectly
83
Q

Blue, CPA, has been asked to render an opinion on the application of accounting principles to a specific transaction by an entity that is audited by another CPA.
Blue may accept this engagement but should

A.  Consult with the continuing CPA to obtain information relative to the transaction.
B.  Report the engagement's findings to the entity's audit committee, the continuing CPA, and management.
C.  Disclaim any opinion that the hypothetical application of accounting principles conforms with generally accepted accounting principles.
D.  Notify the entity that the report is for the restricted use of management and outside parties who are aware of all relevant facts.
A

A. Consult with the continuing CPA to obtain information relative to the transaction.

An accountant is allowed to accept an engagement to provide an opinion on the application of accounting principles to a specific transaction. The accountant, however, must consult with the continuing CPA to obtain all of the available facts pertinent to the transaction.

How well did you know this?
1
Not at all
2
3
4
5
Perfectly
84
Q

In connection with a proposal to obtain a new client, an accountant in public practice is asked to prepare a written report on the application of accounting principles to a specific transaction.
The accountant’s report should include a statement that

A.  Any difference in the facts, circumstances, or assumptions presented may change the report.
B.  The engagement was performed in accordance with Statements on Standards for Consulting Services.
C.  The guidance provided is for management use only and may not be communicated to the prior or continuing auditors.
D.  Nothing came to the accountant's attention that caused the accountant to believe that the accounting principles violated GAAP.
A

A. Any difference in the facts, circumstances, or assumptions presented may change the report.

An accountant’s report on the application of accounting principles to a specific transaction should include: 1) a statement that the engagement was conducted in accordance with applicable AICPA standards, 2) a description of the transaction and the accounting principles to be applied, 3) a statement indicating that responsibility for proper accounting treatment rests with the preparers of the financial statements, and 4) a statement that any difference in the facts, circumstances, or assumptions may change the report. (AU 625)

How well did you know this?
1
Not at all
2
3
4
5
Perfectly
85
Q

Which of the following titles would be considered suitable for financial statements that are prepared on a cash basis?

A.  Income statement.
B.  Statement of operations.
C.  Statement of revenues collected and expenses paid.
D.  Statement of cash flows.
A

C. Statement of revenues collected and expenses paid.

AICPA Professional Standards indicate that use of titles such as balance sheet, statement of financial position, statement of income, statement of operations, and statement of cash flows, or similar titles are usually interpreted as GAAP financial statements. As a result, they should not be used for financial statements prepared in accordance with a special purpose framework. Statement of revenues collected and expenses paid would be considered a suitable title for a cash basis financial statement.

How well did you know this?
1
Not at all
2
3
4
5
Perfectly
86
Q

An entity prepares its financial statements on its income tax basis. A description of how that basis differs from GAAP should be included in the
A. Notes to the financial statements.
B. Auditor’s engagement letter.
C. Management representation letter.
D. Introductory paragraph of the auditor’s report.

A

A. Notes to the financial statements.

The financial statements would contain a footnote that describes the basis of the financial statement presentation and how it differs from GAAP.

How well did you know this?
1
Not at all
2
3
4
5
Perfectly
87
Q

Which of the following items should be included in an auditor’s report for financial statements prepared in conformity with an other comprehensive basis of accounting (OCBOA)?
A. A sentence stating that the auditor is responsible for the financial statements.
B. A title that includes the word “independent.”
C. The signature of the company controller.
D. A paragraph stating that the audit was conducted in accordance with OCBOA.

A

B. A title that includes the word “independent.”

The auditor’s report should include the word “independent,” regardless of the financial reporting framework used in preparing the entity’s financial statements.

How well did you know this?
1
Not at all
2
3
4
5
Perfectly
88
Q

An auditor is engaged to report on selected financial data that are included in a client-prepared document containing audited financial statements. Under these circumstances, the report on the selected data should
A. State that the presentation is a comprehensive basis of accounting other than GAAP.
B. Restrict the use of the report to those specified users within the entity.
C. Refer to the report issued on the entity’s audited financial statements.
D. Indicate that the data are subject to prospective results that may NOT be achieved.

A

C. Refer to the report issued on the entity’s audited financial statements.

How well did you know this?
1
Not at all
2
3
4
5
Perfectly
89
Q

As a condition of obtaining a loan from First National Bank, Maxim Co. is required to submit an audited balance sheet, but not the related statements of income, retained earnings, or cash flows. Maxim would like to engage a CPA to audit only its balance sheet. Under these circumstances, the CPA

A.  May not audit only Maxim's balance sheet if the amount of the loan is material to the financial statements taken as a whole.
B.  May not audit only Maxim's balance sheet if Maxim is a non-issuer.
C.  May audit only Maxim's balance sheet if the CPA disclaims an opinion on the other financial statements.
D.  May audit only Maxim's balance sheet if access to the information underlying the basic financial statements is not limited.
A

D. May audit only Maxim’s balance sheet if access to the information underlying the basic financial statements is not limited.

How well did you know this?
1
Not at all
2
3
4
5
Perfectly
90
Q

Harris, CPA, has been asked to audit and report on the balance sheet of Fox Co., but not on the statements of income, retained earnings, or cash flows. Harris will have access to all information underlying the basic financial statements.
Under these circumstances, Harris may

A.  Not accept the engagement because it would constitute a violation of the profession's ethical standards.
B.  Not accept the engagement because it would be tantamount to rendering a piecemeal opinion.
C.  Accept the engagement because such engagements merely involve limited reporting objectives.
D.  Accept the engagement but should disclaim an opinion because of an inability to apply the procedures considered necessary.
A

C. Accept the engagement because such engagements merely involve limited reporting objectives.

An auditor may report on one basic financial statement and not the others, provided that access to information for all statements is not limited and that all procedures considered necessary are performed.

How well did you know this?
1
Not at all
2
3
4
5
Perfectly
91
Q

Reports are considered special reports when issued in conjunction with
A. Interim financial information reviewed to determine whether material modifications should be made to conform with GAAP.
This would not result in a special report. Instead, a review report on interim financial statements would be issued.
B. Feasibility studies presented to illustrate an entity’s results of operations.
C. Compliance with aspects of regulatory requirements related to audited financial statements.
D. Pro forma financial presentations designed to demonstrate the effects of hypothetical transactions.

A

C. Compliance with aspects of regulatory requirements related to audited financial statements.
Special reports apply to engagements that involve compliance with contracts or regulatory requirements related to financial statements.

How well did you know this?
1
Not at all
2
3
4
5
Perfectly
92
Q

Dunn, CPA, is auditing the financial statements of Taft Co. Taft uses Quick Service Center (QSC) to process its payroll.
Price, CPA, is expressing an opinion on a description of the controls placed in operation at QSC regarding the processing of its customers’ payroll transactions.
Dunn expects to consider the effects of Price’s report on the Taft engagement.
Price’s report should contain a(an)

A.  Description of the scope and nature of Price's procedures.
B.  Statement that Dunn may assess control risk based on Price's report.
C.  Assertion that Price assumes no responsibility to determine whether QSC's controls are suitably designed. A report on controls placed in operation will include the specific statement that the purpose of the engagement was to obtain reasonable assurance about whether the controls were suitably designed to achieve specified control objectives.
D.  Opinion on the operating effectiveness of QSC's internal controls.
A

A. Description of the scope and nature of Price’s procedures.

A report on controls placed in operation includes a description of the scope and nature of the procedures, identification of the party specifying the control objectives, a statement regarding the purpose of the engagement, a disclaimer of opinion on operating effectiveness of the controls, the opinion rendered, a statement regarding inherent limitations, and identification of the parties for whom the report is intended. Note that there are two types of engagements and two types of reports that may be issued:
1) a report on controls placed in operation and
2) a report on controls placed in operation and tests of operating effectiveness.
This question refers to the first type of engagement and report.

How well did you know this?
1
Not at all
2
3
4
5
Perfectly
93
Q

Lake, CPA, is auditing the financial statements of Gill Co.
Gill uses the EDP Service Center, Inc., to process its payroll transactions. EDP’s financial statements are audited by Cope, CPA, who recently issued a report on EDP’s internal control structure. Lake is considering Cope’s report on EDP’s internal control structure in assessing control risk on the Gill engagement.

What is Lake’s responsibility concerning making reference to Cope as a basis, in part, for Lake’s own opinion?

A.  Lake may refer to Cope only if Lake is satisfied as to Cope's professional reputation and independence.
B.  Lake may refer to Cope only if Lake relies on Cope's report in restricting the extent of substantive tests.
C.  Lake may refer to Cope only if Lake's report indicates the division of responsibility.
D.  Lake may not refer to Cope under the circumstances above.
A

D. Lake may not refer to Cope under the circumstances above.

How well did you know this?
1
Not at all
2
3
4
5
Perfectly
94
Q

Green, CPA, is auditing the financial statements of Ajax Co. Ajax uses the DP Service Center to process its payroll. DP’s financial statements are audited by Blue, CPA, who recently issued a report on DP’s policies and procedures regarding the processing of other entity’s transactions. In considering whether Blue’s report is satisfactory for Green’s purposes, Green should
A. Make inquiries concerning Blue’s professional reputation.
B. Assess control risk at the maximum level.
C. Review the audit programs followed by Blue.
D. Perform tests of controls at the DP Service Center.

A

A. Make inquiries concerning Blue’s professional reputation.

Do this first, then review the audit programs followed by Blue.

How well did you know this?
1
Not at all
2
3
4
5
Perfectly
95
Q

When an entity’s auditor issues to an underwriter a comfort letter containing comments on data that have not been audited, the underwriter most likely will receive
A. Negative assurance on capsule information.
B. Positive assurance on supplementary disclosures.
C. A limited opinion on “pro forma” financial statements.
D. A disclaimer on prospective financial statements.

A

A. Negative assurance on capsule information.

In a typical comfort letter, the auditor will provide negative assurance on capsule information.

How well did you know this?
1
Not at all
2
3
4
5
Perfectly
96
Q
Comfort letters ordinarily are signed by the entity's
	A.  Independent auditor.
	B.  Underwriter of securities.
	C.  Audit committee.
	D.  Senior management.
A

A. Independent auditor.

Comfort letters are issued (and signed) by an entity’s independent auditor for the purpose of providing a “due diligence” defense to underwriters and certain other requesting parties in connection with a securities offering.

How well did you know this?
1
Not at all
2
3
4
5
Perfectly
97
Q

Tell, CPA, is auditing the financial statements of Youth Services Co. (YSC), a not-for-profit organization, in accordance with Government Auditing Standards. Tell’s report on YSC’s compliance with laws and regulations is required to contain statements of
Positive assurance Negative assurance
Yes Yes
Yes No
No Yes
No No

A

Positive assurance Negative assurance
Yes Yes

The auditor is required to give positive assurance on the items tested as to compliance with laws and regulations. The auditor provides negative assurance on the items not tested.

How well did you know this?
1
Not at all
2
3
4
5
Perfectly
98
Q

In auditing compliance with requirements governing major federal financial assistance programs under the Single Audit Act, the auditor’s consideration of materiality differs from materiality under generally accepted auditing standards.
Under the Single Audit Act, materiality is

A.  Calculated in relation to the financial statements taken as a whole. Under the Single Audit Act, an audit of compliance with the general and specific requirements applicable to each program should be examined. As a result, materiality is determined separately for each major federal financial program. It is not calculated in relation to the financial statements taken as a whole.
B.  Determined separately for each major federal financial assistance program. The Single Audit Act requires entities receiving federal financial assistance of at least $500,000 to have a single, organization-wide financial and compliance audit. The audit, however, should examine compliance with the general and specific requirements applicable to each program. As a result, materiality is determined separately for each major federal financial assistance program.
C.  Decided in conjunction with the auditor's risk assessment.
D.  Ignored, because all account balances, regardless of size, are fully tested.
A

B. Determined separately for each major federal financial assistance program.

The Single Audit Act requires entities receiving federal financial assistance of at least $500,000 to have a single, organization-wide financial and compliance audit.

The audit, however, should examine compliance with the general and specific requirements applicable to each program. As a result, materiality is determined separately for each major federal financial assistance program.

How well did you know this?
1
Not at all
2
3
4
5
Perfectly
99
Q

Which of the following professional services would be subject to the Statements on Standards for Attestation Engagements (SSAEs)?
A. A management consulting engagement to provide IT-related advice to a client.
B. An engagement to report on an entity’s compliance with statutory requirements.
C. An income tax engagement to prepare federal and state tax returns.
D. The compilation of financial statements from a nonissuer’s accounting records.

A

B. An engagement to report on an entity’s compliance with statutory requirements.

Such an engagement is subject to the SSAEs. Specifically, AT Section 601 applies to engagements related to an entity’s compliance with requirements of applicable laws, regulations, or contracts.

How well did you know this?
1
Not at all
2
3
4
5
Perfectly
100
Q

Mill, CPA, was engaged by a group of royalty recipients to apply agreed-upon procedures to financial data supplied by Modern Co. regarding Modern’s written assertion about its compliance with contractual requirements to pay royalties.
Mill’s report on these agreed-upon procedures should contain a(an)

A.  Disclaimer of opinion about the fair presentation of Modern's financial statements.
B.  List of the procedures performed (or reference thereto) and Mill's findings.
C.  Opinion about the effectiveness of Modern's internal control activities concerning royalty payments.
D.  Acknowledgment that the sufficiency of the procedures is solely Mill's responsibility.
A

B. List of the procedures performed (or reference thereto) and Mill’s findings.

An agreed-upon procedures engagement results in the issuance of a report that identifies the procedures performed and the results obtained. The report would include a list of the procedures performed (or reference thereto) and the findings.

How well did you know this?
1
Not at all
2
3
4
5
Perfectly
101
Q

A CPA’s report on agreed-upon procedures related to management’s assertion about an entity’s compliance with specified requirements should contain
A. A statement of limitations on the use of the report.
B. An opinion about whether management’s assertion is fairly stated.
C. Negative assurance that control risk has not been assessed.
D. An acknowledgment of responsibility for the sufficiency of the procedures.

A

A. A statement of limitations on the use of the report.

How well did you know this?
1
Not at all
2
3
4
5
Perfectly
102
Q

A CPA is required to comply with the provisions of Statements on Standards for Attestation Engagements (SSAE) when engaged to
A. Report on financial statements that the CPA generated through the use of computer software.
B. Review management’s discussion and analysis (MD&A) prepared pursuant to rules and regulations adopted by the SEC.
C. Provide the client with a financial statement format that does not include dollar amounts.
D. Audit financial statements that the client prepared for use in another country.

A

B. Review management’s discussion and analysis (MD&A) prepared pursuant to rules and regulations adopted by the SEC.

How well did you know this?
1
Not at all
2
3
4
5
Perfectly
103
Q
Which of the following is a term for an attest engagement in which a CPA assesses a client's commercial Internet site for predefined criteria that are designed to measure transaction integrity, information protection, and disclosure of business practices?
	A.  ElectroNet.
	B.  EDIFACT.
	C.  TechSafe.
	D.  WebTrust.
A

D. WebTrust.

In a WebTrust assurance engagement, the practitioner expresses an opinion on management’s assertions regarding business practices, transaction integrity, and information protection. The engagement is conducted in accordance with the attestation standards. A WebTrust Seal of Assurance can be displayed for a limited period of time on the websites of entities who receive unqualified opinions in WebTrust engagements.

104
Q

Registered public accounting firms, having the number of issuers as audit clients indicated below, must be inspected how often by the Public Company Accounting Oversight Board?

 More Than 100 Issuers  	 Fewer Than 100 Issuers  
	 Annually 	 Every 2 years 
	 Annually 	 Every 3 years 
	 Every 2 years 	 Every 3 years 
	 Every 2 years 	 Every 5 years
A

Annually Every 3 years

Registered public accounting firms having more than 100 issuers as audit clients must be inspected annually; whereas those having fewer than 100 issuers as audit clients must be inspected every 3 years.

105
Q
The Sarbanes-Oxley Act of 2002 imposes a mandatory rotation applicable to both the audit engagement partner and the quality control (also called review) partner. How long in total is the partner allowed to serve as the engagement partner or review partner before someone else must serve in that capacity?
	A.  3 years.
	B.  5 years.
	C.  7 years.
	D.  10 years.
A

B. 5 years.

Title II of the Sarbanes-Oxley Act of 2002 establishes 5 years as the upper limit for how long someone can serve as the engagement partner or review partner before mandatory rotation is required.

106
Q

The PCAOB issued Auditing Standard #1 in 2004, which specified the reporting language to be used in audit reports applicable to issuers relative to the AICPA standards that the PCAOB adopted on an interim basis in April, 2003. The PCAOB audit report modifications included all of the following, except for

A.  Changed the title of the report from "Independent Auditor's Report" to "Report of the Independent Registered Public Accounting Firm."
B.  Changed the date of the auditor's report to the date that the issuer's financial statements have been filed with the Securities and Exchange Commission.
C.  Replaced the reference to "auditing standards generally accepted in the United States of America" with reference to "the standards of the Public Company Accounting Oversight Board (United States)."
D.  Added a requirement that registered public accounting firms specify their city and state (or country, as applicable) along with their signature and date of the audit report.
A

B. Changed the date of the auditor’s report to the date that the issuer’s financial statements have been filed with the Securities and Exchange Commission.

The PCAOB did not change how the auditor’s report is dated. That date is determined by when the auditor has gathered sufficient appropriate audit evidence as a reasonable basis for the opinion, not by the date of any filings with the SEC. Answers A, C, and D were changes in the auditor’s report as a result of Auditing Standard #1.

107
Q

According to PCAOB auditing standards, after the documentation completion date

A.  Documentation can be added or deleted as deemed appropriate.
B.  Documentation can be added, but not deleted.
C.  Documentation cannot be added, but can be deleted.
D.  Documentation cannot be added or deleted.
A

B. Documentation can be added, but not deleted.

AS #3 states that additional documentation can be added after the documentation completion date (which requires further documentation of the reasons why, etc.), but nothing can be deleted after that date.

108
Q

For audits of public companies, audit documentation must be assembled within how many days of the report release date?

A.  30 days.
B.  45 days.
C.  60 days.
D.  90 days.
A

B. 45 days.

AS #3 specifies a documentation completion date of no more than 45 days following the report release date.

109
Q

The PCAOB issued Auditing Standard #3 to specify the documentation requirements related to audits of public companies. The PCAOB documentation requirements include all of the following, except

A.  All documentation related to client acceptance/retention issues must be maintained at the office primarily responsible for the engagement, so the auditor is prohibited from referring to any materials that might be held at a central repository, such as a national office.
B.  The auditor is required to document all significant findings or issues identified.
C.  For procedures involving inspection of documents, the auditor must document information sufficient to identify the specific items that were tested.
D.  The auditor must document that the underlying accounting records agree to (or reconcile with) the financial statement elements.
A

A. All documentation related to client acceptance/retention issues must be maintained at the office primarily responsible for the engagement, so the auditor is prohibited from referring to any materials that might be held at a central repository, such as a national office.

AS #3 explicitly permits auditors to make appropriate reference to matters documented at a “central repository” (such as independence issues, staffing training matters, and client acceptance/retention issues).

110
Q

According to PCAOB auditing standards, in reporting whether a previously reported material weakness continues to exist, the auditor may appropriately issue

A.  An unqualified or qualified opinion.
B.  An unqualified or adverse opinion.
C.  An unqualified opinion or a disclaimer of opinion.
D.  An unqualified opinion, a qualified opinion, an adverse opinion, or a disclaimer of opinion.
A

C. An unqualified opinion or a disclaimer of opinion.

The auditor is allowed to issue an unqualified opinion or a disclaimer of opinion.

111
Q

According to PCAOB auditing standards, in evaluating whether a material weakness exists, an auditor should focus on materiality at the

A.  Individual account-balance level.
B.  Financial-statement level.
C.  Planning-stage level.
D.  Quantitative level without regard to the qualitative circumstances.
A

B. Financial-statement level.

Paragraph 23 of AS #4 states, “the auditor uses materiality at the financial-statement level, rather than at the individual account-balance level, in evaluating whether a material weakness exists.”

112
Q

According to PCAOB Auditing Standard No. 4, a “stated control objective” is best described as

A.  The specific control objective that management has failed to identify and which, therefore, constitutes a material weakness.
B.  The specific control objective identified by management that, if achieved, would result in the material weakness no longer existing.
C.  A strategic objective of those charged with governance.
D.  The related internal control activities that make it probable that the auditor can assess control risk as low.
A

B. The specific control objective identified by management that, if achieved, would result in the material weakness no longer existing.

Paragraph 16 of AS #4 states, “A stated control objective in the context of an engagement to report on whether a material weakness continues to exist is the specific control objective identified by management that, if achieved, would result in the material weakness no longer existing.”

113
Q

If management has not justified that the alternative accounting principle is preferable to an accounting principle previously used and the effect of the change in accounting principle is material to a company’s financial statements, the auditor should
A. Decide between a qualified opinion and an adverse opinion.
B. Decide between a qualified opinion and a disclaimer of opinion.
C. Issue an unqualified opinion with an explanatory paragraph.
D. Decide between a disclaimer of opinion and an adverse opinion.

A

A. Decide between a qualified opinion and an adverse opinion.

114
Q

An auditor should ordinarily add an explanatory paragraph to the auditor’s report to identify a material matter related to
A. A change in reporting entity resulting from a specific transaction or event.
B. A change in classification in previously issued financial statements.
C. A correction of a material misstatement in previously issued financial statements.
D. All of the above.

A

C. A correction of a material misstatement in previously issued financial statements.

PCAOB Auditing Standard No. 6 identifies 2 specific matters that affect the auditor’s evaluation of consistency of financial statements: (1) a change in accounting principle; and (2) an adjustment to correct a misstatement in previously issued financial statements (i.e., a “restatement”).

115
Q

When there is a change in accounting principle, the auditor should evaluate whether all of the following criteria have been met, except for whether
A. The change has been authorized by those charged with governance.
B. The method of accounting for the effect of the change conforms to GAAP.
C. The disclosures related to the change are adequate.
D. Management has justified that the alternative accounting principle selected is preferable to the previously used accounting principle.

A

A. The change has been authorized by those charged with governance.

116
Q

According to PCAOB auditing standards, the auditor is required to communicate, in writing, all identified material weaknesses to

A.  Management only.
B.  The audit committee only.
    C.  Management and the audit committee.
D.  The appropriate level of personnel, defined as at least one level above where the problem was believed to originate.
A

B. The audit committee only.

AS #5 (paragraph 78) states, “The auditor must communicate, in writing, to management and the audit committee all material weaknesses identified during the audit.”

117
Q

An auditing procedure that is applicable to “testing operating effectiveness” that is not associated with “testing design effectiveness” is

A.  Inquiry.
B.  Observation.
C.  Inspection of relevant documentation.
D.  Reperformance of the control procedure.
A

D. Reperformance of the control procedure.

118
Q

According to PCAOB auditing standards, when the auditor issues separate reports on the financial statements and on internal control over financial reporting,

A.  The reports will normally have different dates, depending upon when audit fieldwork is completed for the financial statements and when the tests of control are completed.
B.  Each report should be entitled "Report of Independent Auditor."
C.  Each report should include a separate paragraph that discusses the "inherent limitations" of any audit engagement.
D.  Each report should include a paragraph that references the other related report.
A

D. Each report should include a paragraph that references the other related report.

119
Q

Which of the following statements is correct regarding characteristics required of an engagement quality reviewer under PCAOB auditing standards?
A. Only a partner of the registered public accounting firm conducting the audit can serve as an engagement quality reviewer.
B. An individual outside of the registered public accounting firm becomes an “associated person” of the registered public accounting firm when receiving compensation from the firm for performing the engagement quality review.
C. There is no requirement that the engagement quality reviewer must be independent from the client involved, since the engagement quality reviewer cannot make engagement team decisions or otherwise assume any responsibilities of the engagement team.
D. The engagement quality reviewer is required to be a partner in a public accounting firm, regardless of whether the reviewer is from within the firm or outside the firm responsible for the audit engagement subject to the engagement quality review.

A

B. An individual outside of the registered public accounting firm becomes an “associated person” of the registered public accounting firm when receiving compensation from the firm for performing the engagement quality review.

AS #7 states, “An outside reviewer who is not already associated with a registered public accounting firm would become associated with the firm issuing the report if he or she … (1) receives compensation from the firm issuing the report for performing the review or (2) performs the review as agent for the firm issuing the report.”

120
Q

PCAOB Auditing Standard No. 12, “Identifying and Assessing Risks of Material Misstatement,” states that the auditor should perform all of the following as risk assessment procedures, except for
A. Incorporating a degree of unpredictability in planned audit procedures.
B. Obtaining an understanding of the company and its environment.
C. Performing analytical procedures.
D. Inquiring of the audit committee, management, and others within the company about the risks of material misstatement.

A

A. Incorporating a degree of unpredictability in planned audit procedures.

The auditor should incorporate a degree of unpredictability in planning audit procedures, but this is considered an “overall response” to the risks of material misstatement, not a “risk assessment procedure.”

121
Q
Each of the following is identified by both the PCAOB risk assessment standards and the AICPA risk assessment standards as an assertion to be specifically addressed by the auditor, except for
	A.  Existence or occurrence.
	B.  Completeness.
	C.  Rights and obligations.
	D.  Presentation and disclosure.
A

C. Rights and obligations.

The PCAOB identified “rights and obligations” as an assertion. Likewise, the AICPA standards (SAS No. 106, “Audit Evidence”) identified “rights and obligations” as one of its 4 assertions associated with “account balances at the period end.”

122
Q
PCAOB Auditing Standard No. 15, "Audit Evidence," identified each of the following as a financial statement assertion to be addressed by the auditor, except for
	A.  Existence or occurrence.
	B.  Completeness.
	C.  Cutoff.
	D.  Presentation and disclosure.
A

C. Cutoff.

“Cutoff” was not identified as one of 5 financial assertions discussed by the PCAOB. The AICPA’s risk assessment standards identify 13 assertions across 3 categories of assertions. And one of those is “cutoff,” which is included among the assertions associated with “transactions or events during the period.”

123
Q

The format and timing of the auditor’s required communication with an issuer’s audit committee is best characterized by the following:
Format of the communication Timing of the communication
Must be in writing Must be by the date of the auditor’s report
Must be in writing Must be prior to issuing the auditor’s report
May be written or oral (unless otherwise specified) Must be by the date of the auditor’s report
May be written or oral (unless otherwise specified) Must be prior to issuing the auditor’s report

A

May be written or oral (unless otherwise specified)
Must be prior to issuing the auditor’s report

Unless otherwise specified, the communication may be written or oral. (For example, an engagement letter obviously must be in writing.) The PCAOB requires that the communication be timely, and prior to the issuance of the auditor’s report.

124
Q

Under PCAOB auditing standards, the auditor should communicate all of the following matters to an issuer’s audit committee at the beginning of the audit engagement, except for
A. Significant issues that the auditor discussed with management in connection with the auditor’s appointment/retention.
B. The terms of the engagement, including the objectives of the audit and the parties’ respective responsibilities.
C. The qualitative aspects of the entity’s significant accounting policies, including any indications of management bias.
Although the auditor would discuss with the audit committee the qualitative aspects of the entity’s significant accounting policies (and any indications of management bias) those matters would be discussed at the end of the audit.
D. An overview of the audit strategy and timing of the audit, along with any significant risks identified by the auditor.

A

C. The qualitative aspects of the entity’s significant accounting policies, including any indications of management bias.

Although the auditor would discuss with the audit committee the qualitative aspects of the entity’s significant accounting policies (and any indications of management bias) those matters would be discussed at the end of the audit.

125
Q

The objectives of Auditing Standard No. 16, “Communications with Audit Committees,” include all of the following, except for
A. Enhancing communications between the audit committee and the entity’s internal audit function.
B. Obtaining information from the audit committee that is relevant to the audit.
C. Providing the audit committee with timely information about significant audit issues.
D. Communicating to the audit committee the auditor’s responsibilities and establish an understanding of the terms of the engagement.

A

A. Enhancing communications between the audit committee and the entity’s internal audit function.

126
Q

The auditor’s report on supplemental information should include a statement about each of the following, except for
A. A statement that the supplemental information is management’s responsibility.
B. A statement that the methods of measurement and presentation have not changed from those used in the prior period.
C. A statement that the supplemental information complies with the applicable regulatory requirements.
D. An opinion (or disclaimer) as to whether the supplemental information is fairly stated in relations to the financial statements as a whole.

A

B. A statement that the methods of measurement and presentation have not changed from those used in the prior period.

The auditor should obtain management’s representation that the methods of measurement or presentation have not changed from those used in the prior period. However, the auditor’s report does not include such a statement.

127
Q

The definition of “supplemental information” under PCAOB Auditing Standard No. 17 includes all of the following, except for
A. A public company’s sustainability report consisting of a variety of financial and nonfinancial measures of performance, which is made available to readers on the entity’s web site.
B. Supporting schedules that brokers and dealers are required to file with the Securities and Exchange Commission.
C. Information outside of the financial statements that is derived from the entity’s accounting records, which is covered by the auditor’s report in relation to financial statements audited under PCAOB auditing standards.
D. Information that is required to be presented under the rules of a regulatory authority, which is covered by the auditor’s report in relation to financial statements audited under PCAOB auditing standards.

A

A. A public company’s sustainability report consisting of a variety of financial and nonfinancial measures of performance, which is made available to readers on the entity’s web site.

Information that a company voluntarily presents on its web site is outside the PCAOB’s definition of “supplemental information.” As a practical matter, sustainability reports are not required, nor is third-party assurance on sustainability required.

128
Q

Suppose that management of an issuer makes an assertion in a footnote to the company’s financial statements that material transactions with related parties were conducted on terms equivalent to those prevailing in arm’s-length transactions. If evidence cannot be obtained to support this assertion and management declines to alter that footnote, what type of audit opinion would be appropriate?
A. Unqualified with an explanatory paragraph.
B. Qualified for a scope limitation.
C. Qualified or adverse for a material misstatement.
D. Disclaimer of opinion.

A

C. Qualified or adverse for a material misstatement.

AS No. 18 states that the auditor should consider a qualified or adverse opinion under such circumstances.

129
Q

For any related-party transaction that is required to be disclosed or that is determined to be a significant risk, PCAOB Auditing Standard No. 18 (Related Parties) requires the auditor to do each of the following, except
A. Evaluating the financial capability of the related parties with respect to their significant responsibilities in connection with the transaction.
B. Ascertaining that the transaction has been authorized and approved in accordance with the company’s established policies.
C. Reading applicable underlying documents for consistency with explanations about the business purpose obtained from inquiries and other procedures performed.
D. Obtaining a written representation from the company’s lawyer that the related-party transaction complies with all applicable laws and regulations.

A

A. Evaluating the financial capability of the related parties with respect to their significant responsibilities in connection with the transaction.

Under these circumstances, AS No. 18 requires the auditor to evaluate the financial capability of the related parties with respect to their significant responsibilities.

130
Q

Which of the following statements best explains why the CPA profession has found it essential to promulgate ethical standards and to establish means for ensuring their observance?
A. A distinguishing mark of a profession is its acceptance of responsibility to the public.
B. A requirement for a profession is to establish ethical standards that stress primary responsibility to clients and colleagues.
C. Ethical standards that emphasize excellence in performance over material rewards establish a reputation for competence and character.
D. Vigorous enforcement of an established code of ethics is the best way to prevent unscrupulous acts.

A

A. A distinguishing mark of a profession is its acceptance of responsibility to the public.

131
Q

Which of the following is true regarding the Principles of Professional Conduct?
A. To live up to the Code of Professional Conduct, members may have to work hard, but they do not have to sacrifice their own best interests.
B. Members must not only be competent in the provision of professional services; they must also cooperate with other members to improve the art of accounting.
C. Due care in the audit area is satisfied if a member knows generally accepted accounting principles and generally accepted accounting standards inside and out.
D. Because the Code of Professional Conduct does not expressly prohibit a member from moonlighting as a circus trapese performer, a member could perform at a local bar as “Sam the Flying CPA.”

A

B. Members must not only be competent in the provision of professional services; they must also cooperate with other members to improve the art of accounting.

132
Q

Which of the following is not a type of threat to compliance that is featured in the Conceptual Framework?
A. Adverse interest threats.
B. Espionage threats.
The code does not speak about espionage threats.
C. Familiarity threats.
D. Undue influence threats.

A

B. Espionage threats.

The code does not speak about espionage threats.

133
Q

Brecken wishes to certify financial statements that materially depart from GAAP. She believes that it is necessary to do so in order to prevent the statements from being misleading in light of a new federal statute, which GAAP has not been amended to accommodate. In order to implement this departure, Brecken should do which of the following:
A. Describe the departure from GAAP.
B. Describe the departure’s approximate effects.
C. State the reasons why compliance with GAAP would mislead.
D. All three choices provided.

A

D. All three choices provided.

According to Rule 203.

134
Q

Dena prepared personal income tax returns for 100 clients last year. Two of her clients have come to her with valid complaints about errors she made in their returns, causing them to pay more than they should have in taxes. Which of the following is true?
I. Dena’s CPA license should be revoked, because she obviously cannot exercise due professional care.
II. While it is certainly possible that Dena has acted without due professional care in these two instances, the AICPA does not demand perfection.
A. I only.
B. II only.
C. I and II.
D. Neither I nor II.

A

B. II only.

The AICPA and state accountancy boards do not expect CPAs to be “Ms. or Mr. Super Accountant.”

135
Q
Members may properly:
I.	Advocate on behalf of audit clients.
II.	Advocate on behalf of tax clients.
	A.  I only.
	B.  II only.
Members may advocate on behalf of tax and advisory service clients, although they should never stretch the bounds of performance standards, go beyond sound and reasonable professional practice, or compromise their credibility.
	C.  I and II.
	D.  Neither I nor II.
A

B. II only.

Members may advocate on behalf of tax and advisory service clients, although they should never stretch the bounds of performance standards, go beyond sound and reasonable professional practice, or compromise their credibility.

136
Q

Which of the following is true?
A. Clients have the right to veto any outsourcing by a CPA firm.
B. Clients must be informed in writing before any professional services are outsourced.
C. If the client objects to outsourcing of professional services, then the member should either not outsource the work or decline to provide the service altogether.
D. All three choices provided.

A

C. If the client objects to outsourcing of professional services, then the member should either not outsource the work or decline to provide the service altogether.

The outsourcing of professional services requires notification and client approval.

137
Q

To exercise due professional care, an auditor should
A. Critically review the judgment exercised by those assisting in the audit.
B. Examine all available corroborating evidence supporting management’s assertions.
C. Design the audit to detect all instances of illegal acts.
D. Attain the proper balance of professional experience and formal education.

A

A. Critically review the judgment exercised by those assisting in the audit.

Using due care requires that a CPA take those steps that are reasonably necessary to ensure the accuracy of an audit. If others are assisting in the audit, the accountant in charge has a duty to look over their work, as he/she is ultimately responsible for it.

138
Q

Which of the following statements is correct with respect to ownership, possession, or access to a CPA firm’s audit documentation?
A. Audit documentation is subject to the privileged communication rule which, in most jurisdictions, prevents any third-party access to the audit documentation.
B. Audit documentation may never be obtained by third parties unless the client consents.
C. Audit documentation is a client’s exclusive property.
D. Audit documentation is not transferable to a purchaser of a CPA practice unless the client consents.

A

D. Audit documentation is not transferable to a purchaser of a CPA practice unless the client consents.

Third parties may not always get access to the audit documentation, but may have access in three situations:

  1. If the client consents, they may be released.
  2. If a court orders their release.
  3. If released to a CPA society’s quality review board.
139
Q

Which of the following actions by a CPA most likely does not constitute an act discreditable to the profession?
A. Sexually harassing an employee.
B. Providing for a right of contribution from the client in an audit engagement letter.
C. Providing for a right of indemnification from the client in an audit engagement letter.
D. All of the above.

A

B. Providing for a right of contribution from the client in an audit engagement letter.

While the SEC and most government agencies oppose full indemnification of wrongdoers, contribution is typically allowed and, therefore, would not violate Rule 501.

140
Q

Assuming appropriate disclosure is made, which of the following fee arrangements generally would be permitted under the ethical standards of the profession?

A.  A fee paid to the client's audit firm for recommending investment advisory services to the client.
B.  A fee paid to the client's tax accountant for recommending a computer system to the client.
C.  A contingent fee paid to the CPA for preparing the client's amended income tax return.
D.  A contingent fee paid to the CPA for reviewing the client's financial statements.
A

B. A fee paid to the client’s tax accountant for recommending a computer system to the client.

Tax accountants can accept referral fees and commissions. However, they should be disclosed to the client (and the question instructs us to assume appropriate disclosure).

141
Q

Tammy is looking to increase the revenue stream for her accounting firm. She is thinking of using commissions and referral fees to do so. Which of the following is true regarding commissions and referral fees?
I. Neither is permitted when the client is an audit client.
II. Both are permitted when the client is not an audit client, if they are properly disclosed.
A. I only.
B. II only.
C. I and II.
D. Neither I nor II.

A

C. I and II.

Because both I and II are accurate, this is the best answer.

142
Q
Young is an audit partner and a CPA at a Big Four firm. Young has invested in a local firm, LMN, that provides computer and other technical support for businesses. Young owns a majority interest in LMN. Which of the following must observe AICPA rules on commissions and referral fees regarding an LMN client that is also an audit client of the Big Four firm?
	A.  Young.
	B.  LMN.
	C.  LMN employees.
	D.  All three choices provided.
A

D. All three choices provided.

143
Q

Tondry is a CPA working for a Big Four firm as an auditor. Tondry has invested in a local firm, XYZ, that provides technical computer support and other nonaudit services for businesses. At least one of the businesses that receives technical support from XYZ (and purchases software on XYZ’s recommendation for which XYZ received a commission), is an audit client for which Tondry is on the audit engagement team. Given these facts, which of the following is true?
I. There is clearly an independence problem here.
II. There is an independence problem here only if Tondry (or Tondry with his firm or other firm members) controls XYZ.
A. I only.
B. II only.
C. I and II.
D. Neither I nor II.

A

B. II only.

While Tondry must, of course, abide by all professional rules, the firm in which he has invested (XYZ) and its employees must do so only if controlled by Tondry.

144
Q

Which firms must have a majority of their financial interests owned by CPAs?
I. Attest firms.
II. Firms that identify themselves as “Members of the AICPA.”

    A.  I only.
B.  II only.
C.  I and II.
D.  Neither I nor II.
A

A. I only.

Attest firms must be majority owned by CPAs.

145
Q

A CPA is permitted to disclose confidential client information without the consent of the client to
I. Another CPA firm if the information concerns suspected tax return irregularities.

II. A state CPA society voluntary quality control review board.

A.  I only.
B.  II only.
C.  Both I and II.
D.  Neither I nor II.
A

B. II only.

A CPA may generally disclose confidential information without a client’s consent if it is necessary to avoid violating GAAP, if in response to an ethics inquiry by a quality review board, or pursuant to a court order.

146
Q

Which of the following statements concerning an accountant’s disclosure of confidential client data is generally correct?
A. Disclosure may be made to any state agency without a subpoena.
B. Disclosure may be made to any party on consent of the client.
Disclosure may not be made to any party without either a court order or the consent of the client, unless the requesting party is a state CPA regulatory body. If the client consents, then the information may be released to anyone that the client has approved.
C. Disclosure may be made to comply with an IRS audit request.
D. Disclosure may be made just for the heck of it.

A

B. Disclosure may be made to any party on consent of the client.

Disclosure may not be made to any party without either a court order or the consent of the client, unless the requesting party is a state CPA regulatory body. If the client consents, then the information may be released to anyone that the client has approved.

147
Q

A CPA in public practice may not disclose confidential client information regarding auditing situations without the client’s consent in response to which of the following situations?

A.  A review of the CPA's professional practice by a state CPA society.
B.  A letter to the client from the IRS.
C.  An inquiry from the professional ethics division of the AICPA.
D.  A court-ordered subpoena or summons.
A

B. A letter to the client from the IRS.

A CPA should refuse to produce confidential client records in response to a mere request from the IRS or SEC.

148
Q

A CPA’s audit documentation
A. Need not be disclosed under a federal court subpoena.
B. Must be disclosed under an IRS administrative subpoena.
C. Must be disclosed to another accountant purchasing the CPA’s practice even if the client hasn’t given permission.
D. Need not be disclosed to a state CPA society quality review team.

A

B. Must be disclosed under an IRS administrative subpoena.

There are only a few circumstances in which information may be disclosed without the consent of the client. These include when a valid court order demanding release of the information is issued, when a quality review board of a state’s CPA society requests the information, when a client consents to disclosure, and whenever professional obligations otherwise require it.

149
Q

Page, CPA, has T Corp. and W Corp. as audit clients. T Corp. is a significant supplier of raw materials to W Corp. Page also prepares individual tax returns for Time, the owner of T Corp., and West, the owner of W Corp. When preparing West’s return, Page finds information that raises going-concern issues with respect to W Corp.
May Page disclose this information to Time?

A.  Yes, because Page has a fiduciary relationship with Time.
B.  Yes, because there is no accountant-client privilege between Page and West.
C.  No, because the information is confidential and may not be disclosed without West's consent.
D.  No, because the information should only be disclosed in Page's audit report on W Corp.'s financial statements.
A

C. No, because the information is confidential and may not be disclosed without West’s consent.

150
Q

Which of the following acts by a CPA is a violation of professional standards regarding the confidentiality of client information?

A.  Releasing financial information to a local bank with the approval of the client's mail clerk.
B.  Allowing a review of professional practice without client authorization.
C.  Responding to an enforceable subpoena.
D.  Faxing a tax return to a loan officer at the request of the client.
A

A. Releasing financial information to a local bank with the approval of the client’s mail clerk.

It is extremely unlikely that a client’s mail clerk would have authority to approve the release of confidential financial information. The CPA, therefore, should not release it.

151
Q

The Melancon accounting firm audits XYZ Co. out of its Boston office. Who, among the following Melancon employees, is a “covered member” regarding XYZ?
A. Sim, the receptionist who handles many administrative tasks for the XYZ audit team.
B. Tim, the records guy who lugs around many, many boxes of records that the audit team must review.
C. Bit, the new college accounting graduate, who was added to the XYZ audit team on his first day at Melancon.
D. None of the three choices provided.

A

C. Bit, the new college accounting graduate, who was added to the XYZ audit team on his first day at Melancon.

As a staff member who works on the engagement, Bit is a “covered member.”

152
Q
Network firms are firms that cooperate to enhance their capabilities to provide professional services and share one or more of the following except:
	A.  A common brand.
	B.  A common bathroom.
	C.  Common control.
	D.  Professional resources.
A

B. A common bathroom.

153
Q

Which of the following is (are) true?
A. If Firm A and Firm B are in the same network, A must be independent of B’s attest clients if use of the audit or review reports is unrestricted.
B. If Firm A and Firm B are in the same network, A must be independent of B’s attest clients if use of the audit or review reports is restricted.
C. Firms may never use partners or professional employees of other firms on their audit teams.
D. All three answer choices provided are true.

A

A. If Firm A and Firm B are in the same network, A must be independent of B’s attest clients if use of the audit or review reports is unrestricted.

Independence is required if use of such reports is unrestricted.

154
Q

The Cheng Accounting Firm is concerned with litigation costs, so it is inserting provisions in all its engagement letters that require arbitration rather than litigation of disputes between Cheng and its attest clients. Which of the following is true?
I. Such a provision is simply not allowed by the code.
II. Such a provision is allowed, but if it is invoked, Cheng should apply the Conceptual Framework to determine whether independence is impaired by the fact that it and its client have potentially been placed in positions of material adverse interests.
A. I only.
B. II only.
C. Both I and II.
D. Neither I nor II.

A

B. II only.

The code does note that entering into binding arbitration potentially creates independence problems by placing the firm and the client in positions of material adverse interests, creating a self-interest threat.

155
Q

The Fritz Accounting Firm finished an audit of ABC Co. nine months ago and then fired ABC as a client. Fritz was independent of ABC when it signed off on the audit. It is not independent of ABC now, but ABC is attempting to borrow a large sum of money and ABC would like to show the audited financial statements to the borrower with Fritz’s permission. Fritz generally wishes to accommodate ABC but would not like to allow this to be done if Fritz’s financial situation has dramatically worsened in the last nine months. Which of the following is not true?
A. This is fine so long as Fritz substantially reperforms the audit to ensure that the numbers are still valid today.
B. If Fritz merely makes inquiries of ABC’s new auditor to reassure itself, there should be no independence problem.
C. If Fritz simply reads subsequent financial statements issued by ABC, there should be no independence problem.
D. All three choices provided.

A

A. This is fine so long as Fritz substantially reperforms the audit to ensure that the numbers are still valid today.

Because Fritz is no longer independent, it should not be performing an essentially new audit.

156
Q

A CPA purchased stock in a client corporation and placed it in a trust as an educational fund for the CPA’s minor child. The trust securities are not material to the CPA’s wealth but are material to the child’s personal net worth. According to the AICPA Code of Professional Conduct, would this action impair the CPA’s independence with the client?

A.  No, because the CPA would not have a direct financial interest in the client.
B.  Yes, because the stock would be a direct financial interest and materiality is a factor.
     C.  Yes, because the stock would be an indirect financial interest and materiality is not a factor.
D.  Yes, because the stock would be a direct financial interest and materiality is not a factor.
A

D. Yes, because the stock would be a direct financial interest and materiality is not a factor.

This is a direct financial interest. Therefore, materiality is not a factor. A direct financial interest impairs independence even if it is immaterial.

157
Q

Exam Results
Question #1 (AICPA.141031AUD-SIM)
Which of the following creates an independence problem?
I. Sally participates in her firm’s retirement plan, which allows her to select which companies’ stocks go into her portfolio. Sally selects a small amount of ABC Co. stock, even though she is on her firm’s audit team for ABC.
II. Although Sally has not selected ABC Co. stock for her retirement plan’s portfolio, her spouse, Joe, has done so.
A. I only.
B. II only.
C. Both I and II.
D. Neither I nor II.

A

C. Both I and II.

Both of these situations create an independence problem. The ability of Sally and Joe to self-direct their investments makes their interests in ABC direct. Therefore, there is an independence problem even if the investments are immaterial.

158
Q

Which of the following is true?
A. A covered member’s financial interest in a mutual fund is a direct interest in the fund.
B. A covered member’s financial interest in a mutual fund is an indirect interest in the fund.
C. A covered member’s financial interest in a diversified mutual fund is a direct interest in the fund’s underlying investments.
D. A covered member’s financial interest in an undiversified mutual fund is a direct interest in the fund’s underlying investments.

A

A. A covered member’s financial interest in a mutual fund is a direct interest in the fund.

159
Q

Art is on his firm’s audit team for client ABC Co. Which of the following is an indirect financial interest?
A. Art is a general partner in a partnership that owns stock in ABC.
B. Art is a limited partner in a partnership that owns stock in ABC and is on the partnership’s investments committee.
C. Art is a member in a member-managed limited liability corporation that owns stock in ABC.
D. Art is a member in an agent-managed limited liability corporation that owns stock in ABC.

A

D. Art is a member in an agent-managed limited liability corporation that owns stock in ABC.

Unless Art participates in the investment decisions of his limited liability corporation, which is not indicated in the facts, his interests in its underlying investments are indirect if the LLC is agent managed.

160
Q

Which of the following situations creates an independence problem for Kim?
I. Kim owns an insurance policy that does not contain an investment option and was issued under normal terms, procedures, and requirements.
II. Kim owns an insurance policy with an investment option, and she invested a small amount in the ABC Mutual Fund, even though Kim is a covered member for purposes of ABC, which is an audit client of her firm.
A. I only.
B. II only.
C. Both I and II.
D. Neither I nor II.

A

B. II only.

Under these facts, Kim’s interest in ABC is a direct financial interest, which creates an independence problem.

161
Q

The Patton Accounting Firm and one of its partners, Tilly, have depository accounts at the ABC Bank. ABC has just approached Patton about becoming the bank’s auditor. In which of the following situations would there be an independence problem if Patton became ABC’s auditor?
A. ABC is in robust financial health.
B. Tilly’s account is fully insured.
Tilly may maintain a depository account at ABC so long as it is fully insured.
C. Tilly’s account is not fully insured, but the uninsured amount is not material to her financial situation.
D. None of the three choices provided

A

D. None of the three choices provided.

162
Q

Tim is a covered member regarding his firm’s attest client, ABC Brokerage Co. Tim maintains a brokerage account at ABC. Which of the following must be true for there not to be an independence problem here?
I. Tim’s account is handled according to ABC’s normal terms, procedures, and requirements.
II. Tim’s assets at risk in the ABC account are immaterial to his net worth.
A. I only.
B. II only.
C. Both I and II.
D. Neither I nor II.

A

C. Both I and II.

163
Q

On June 1, 20x8, a CPA obtained a $100,000 personal loan from a financial institution client for whom the CPA provided compilation services. The loan was fully secured and considered material to the CPA’s net worth. The CPA paid the loan in full on December 31, 20x9. On April 3, 20x9, the client asked the CPA to audit the client’s financial statements for the year ended December 31, 20x9. Is the CPA considered independent with respect to the audit of the client’s December 31, 20x9 financial statements?
A. No, because the CPA had a loan with the client during the period of a professional engagement.
If the proper conditions are met, independence is not impaired if a CPA has a loan from a financial institution client during the period of a professional engagement.
B. No, because the CPA had a loan with the client during the period covered by the financial statement.
C. No, because secured loans are not allowed from lending institution attest clients.
D. Yes, because the conditions needed to prevent a loan from violating independence rules appear to be met here.

A

D. Yes, because the conditions needed to prevent a loan from violating independence rules appear to be met here.

The general rule is that independence is impaired if a CPA borrows money from a client. However, there are exceptions. One exception exists where a secured loan, such as we have here, comes from a financial institution client (also the case here) and four safeguards are met. First, the loan must be obtained under the lending institution’s normal procedures and terms, which appears to be the case here. Second, the loan must have been obtained prior to the lending institution becoming an attest client (or pursuant to some other exceptions), which is also the case here. Third, the loan must be kept current. Fourth, the estimated fair value of the collateral must exceed the outstanding balance during the term of the loan. Although the facts of the question do not tell us explicitly if the third and fourth conditions were met here, the fact that the loan was paid off in December of 20x9 hints that they were.

164
Q

Which of the following situations does not create an independence problem for the Brixton Accounting Firm relative to its attest client ABC Co.
A. Both Brixton and ABC Co. own Microsoft stock.
B. Both Brixton and ABC Co. purchase major stakes in a local technology start-up company.
C. One of Brixton’s covered members is friends with ABC’s chief executive officer, and they pursue their passion for skiing by jointly purchasing a ski chalet in Vail.
D. None of the above create an independence problem.

A

A. Both Brixton and ABC Co. own Microsoft stock.

Because Microsoft is a public company, this situation creates no independence problem, although it might be considered a joint investment.

165
Q
Which of the following is not a close relative?
	A.  A parent.
	B.  A spousal equivalent.
	C.  A sister.
	D.  A nondependent child.
A

B. A spousal equivalent.

Close relatives include parents, siblings, and nondependent children but not spousal equivalents, who are immediate family members.

166
Q

Anchorage Alltime Dairy (AAD) is an audit client of Thorn Granton’s Juneau office. Al is on the audit team. Betsy evaluates Al’s work and determines his pay. Carol is a tax professional in the Juneau office, who provided 20 hours of tax advice to AAD during the last audit cycle. Dave is a tax partner in the Juneau office, who has never provided any services to AAD. Which of the following situations would create an independence problem under the new AICPA rules?
A. Al’s brother Sam is in charge of manure disposal for AAD.
B. Betsy’s live-in lover owns enough AAD stock that it is material to him, but it does not allow him to exert significant influence over AAD.
C. Carol’s mother is a receptionist for AAD and, via its pension plan, indirectly owns a small amount of AAD stock.
D. None of the choices provided.

A

B. Betsy’s live-in lover owns enough AAD stock that it is material to him, but it does not allow him to exert significant influence over AAD.

Betsy is in a position to influence the attest engagement team. Her live-in lover is a spousal equivalent, which qualifies him as an immediate family member. He is therefore covered by the independence rules. Although he could hold a non-key position, if he owns enough AAD stock that it is material to him, there is clearly a significant independence problem.

167
Q

RJ is a partner at PricewaterhouseCoopers’ New York office. His engagement team is currently auditing PwC’s largest client, IBM. Which of the following conditions would not violate AICPA independence guidelines?
A. RJ’s father, Lou, is the CEO of Exxon Mobil. Lou also serves as the chair of IBM’s audit committee.
B. RJ owns 100 shares of AIM Energy A, a mutual fund trading on the Nasdaq. This mutual fund is comprised of several stocks, including IBM.
C. RJ’s dependent stepson, CJ, owns 1% of IBM’s outstanding shares, acquired through his deceased father’s will. RJ knows of this interest.
D. RJ’s wife, Sue, is employed with Protiviti, a firm that provides internal audit services for public companies. She is currently providing internal audit services for IBM.

A

B. RJ owns 100 shares of AIM Energy A, a mutual fund trading on the Nasdaq. This mutual fund is comprised of several stocks, including IBM.

RJ’s ownership of AIM Energy A is both immaterial and indirect, which is permitted by the AICPA.

168
Q

Tisha is a partner at a local accounting firm, which provides audit services for the United Way. She is not on the engagement team, but is, obviously (as partner in the firm), in a position to influence the audit team’s findings and decisions. Tisha also serves as an honorary trustee on the United Way Board of Trustees. As an honorary trustee, she does not take any management role nor make any decisions, and all information brochures and pamphlets circulated by the United Way indicate her honorary position. Which of the following is true regarding whether there is an independence problem here?
I. Yes—As partner in the firm where the lead audit partner performs the audit of the United Way, Tisha is considered a “covered member” and is required to follow independence rules.
II. Yes—Because Tisha is a partner in the accounting firm that provides attestation services to the United Way, she cannot serve as a trustee of that organization.
III. No—Tisha serves as an honorary trustee of the United Way and exerts no influence over management and its decisions related to the organization.
A. I only.
B. II only.
C. III only.
D. Both I and II.

A

C. III only.

So long as Tisha follows the guidelines, she can serve on the honorary board of an audit client without creating independence problems.

169
Q

Under the ethical standards of the profession, which of the following business relationships would generally not impair an auditor’s independence?
A. Promoter of a client’s securities.
B. Member of a client’s board of directors.
C. Client’s general counsel.
D. Advisor to a client’s board of trustees.

A

D. Advisor to a client’s board of trustees.

The role of advisor to a client’s board is not forbidden by AICPA independence rules.

170
Q
GMPK audits Wonderwear Corporation, a public company. The audit cycle runs from May 15 to May 15. Sandy was an important member of GMPK's engagement team for Wonderwear. She quit GMPK on June 1, 2009, intent upon taking a job as second in command to the CFO for Wonderwear. When may Sandy assume these duties?
	A.  June 2, 2009.
	B.  May 15, 2010.
	C.  May 15, 2011.
	D.  All of the choices provided.
A

C. May 15, 2011.

By waiting until May 15, 2011, Sandy should not have an independence issue because an entire audit cycle (May 15, 2010 to May 15, 2011) has passed since she left GMPK.

171
Q
A cooling-off period of how many years is required before a member of an issuer's audit engagement team may begin working for the registrant in a key position?
	A.  One year.
	B.  Two years.
	C.  Three years.
	D.  Four years.
A

A. One year.

The cooling-off period is one year, so this answer is correct.

172
Q

Which of the following situations would likely cause an independence problem?
A. The Diko Accounting Firm audits the financial statements of the Toledo Country Club. Sal, a covered member at Diko, joins the country club because she loves to play tennis.
So long as Sal’s relationship with the country club is primarily social, there is no independence problem.
B. Diko audits the financial statements of the American Federation of Tax Evaders. Tom, who is not a covered member but is a professional employee of Diko, also serves on the board of directors of the federation.
C. Diko audits the financial statements of the Southwest Louisville Credit Union. Mit, a covered member at Diko, is a member of the credit union by virtue of where he lives.
D. None of the three choices provided.

A

B. Diko audits the financial statements of the American Federation of Tax Evaders. Tom, who is not a covered member but is a professional employee of Diko, also serves on the board of directors of the federation.

If a partner or professional employee of a firm is employed by or associated in an important role, such as director, with an attest client trade association, there is a significant threat to independence.

173
Q

Rachel is on her firm’s attest team for client ABC Co. Which standards apply to gifts and entertainment?
A. If Rachel receives a gift from ABC, the key question is whether or not it was “reasonable in the circumstances.”
B. If Rachel receives entertainment from ABC, the key question is whether or not it was “clearly insignificant” to Rachel.
C. If Rachel receives a gift from ABC, the key question is whether or not it was “clearly insignificant” to Rachel.
D. None of the three choices provided.

A

C. If Rachel receives a gift from ABC, the key question is whether or not it was “clearly insignificant” to Rachel.

The code applies the “clearly insignificant” standard to gifts and the “reasonable in the circumstances” standard to entertainment.

174
Q
Which categories of covered members must be concerned with independence problems that might arise from gifts they receive from attest clients?
I.	Team members.
II.	Other partners in the office.
	A.  I only.
	B.  II only.
	C.  Both I and II.
	D.  Neither I nor II.
A

A. I only.

Regarding independence, the firm, team members, and those in a position to influence may not accept gifts from attest clients unless the value of those gifts is clearly insignificant to the recipient.

175
Q

Code provisions, rather than Sarbanes-Oxley provisions, will govern provision of nonaudit services when:
I. An auditor provides nonaudit services to a public company audit client.
II. An auditor provides nonaudit services to a private company audit client.
III. An auditor provides nonaudit services to a public company that is not an audit client.
A. I only.
B. II only.
C. I and III.
D. II and III.

A

D. II and III.
Code provisions, rather than SOX, govern the provision of nonaudit services to a private company audit client and to a public company that is not an audit client.

176
Q

The Flakel Accounting Firm audits ABC Co., a public company. ABC would like to fire its current tax consultant and replace it with Flakel. Which of the following is true?
I. This action would necessarily violate the Code.
II. This action would violate Sarbanes-Oxley rules, unless ABC’s audit committee preapproved the hiring.
III. This action would violate Sarbanes-Oxley rules even if ABC’s audit committee preapproved the hiring.
A. I only.
B. II only.
C. III only.
D. I and II.

A

B. II only.

Sarbanes-Oxley requires audit committee preapproval.
The code does not prevent auditors from providing tax consulting services to their attest clients so long as the general requirements are met.

177
Q

Which of the following benefit plan administration services may not be provided by an auditor to a private company attest client?
A. Communicate summary plan data to plan trustee.
B. Advice client management regarding impact of plan provisions.
C. Prepare account valuations.
D. Make disbursements on plan’s behalf.

A

D. Make disbursements on plan’s behalf.

This is a management responsibility that goes too far to avoid an independence problem.

178
Q

Which of the following is not a duty owed by a member in business?
A. To be candid and truthful when communicating with an employer’s external auditor.
B. To correct inaccurate financial statements or entries.
This is an important duty owed by a member in business.
C. To be independent in fact and appearance.
D. To decline gifts or entertainment that would violate an employer’s rules or be unreasonable in the circumstances.

A

C. To be independent in fact and appearance.

Members in public practice have a duty of independence, but members in business do not.

179
Q

Which of the following is not true regarding application of the Conceptual Framework for members in business?
A. An adverse interest threat arises when a member in business sues her employer.
B. A familiarity threat arises when a member in business hires a relative to work for his employer.
C. A self-review threat arises when a member in business reviews some internal audit work that she herself performed before she was promoted to her current position.
D. A familiarity threat arises when a member in business has a long association with an employer.

A

D. A familiarity threat arises when a member in business has a long association with an employer.

180
Q

Which of the following is a duty owed by an “other member”?
I. Duty to be independent in fact and appearance.
II. Duty to avoid committing discreditable acts.
A. I only.
B. II only.
C. I and II.
D. Neither I nor II.

A

B. II only.

Even other members must avoid discreditable acts, but don’t have to be independent in fact and appearance (and neither do members in business).

181
Q

Which of the following nonaudit services may PCH provide to a public company audit client with preapproval from the client’s audit committee?
A. Financial information systems design and implementation.
B. Internal audit outsourcing services.
C. Human resources services.
SOX clearly prohibits an auditor from providing human resources services to a public company audit client.
D. General tax planning.

A

D. General tax planning.

If preapproved by the client’s audit committee, general tax services may be provided by an auditor to a public company audit client. The client must also disclose the services’
provision in periodic filings with the SEC.

182
Q
According to the Sarbanes-Oxley Act of 2002, what is the maximum number of years an audit partner can perform audit services for an issuer before the auditor rotation is required?
	A.  2 years.
	B.  3 years.
	C.  4 years.
	D.  5 years.
A

D. 5 years.

183
Q

An auditor may provide an issuer client any of the following non-audit services without impairing independence and without obtaining the preapproval of the audit committee, except
A. Non-audit services with revenues in aggregate of less than 5% of the total revenues paid by the issuer to the auditor during the fiscal year in which the non-audit services are provided.
B. Non-audit services that were promptly brought to the attention of, and approved by, the audit committee prior to the completion of the audit.
C. Non-audit services to perform financial information systems design and implementation.
D. Services that the issuer did not recognize as non-audit services at the time of the engagement.

A

C. Non-audit services to perform financial information systems design and implementation.

SOX lists several nonaudit services that audit firms may not provide for their public company audit clients, and this is near the top of the list.

184
Q

Which of the following is true regarding the relationship of the SEC and the PCAOB?
A. The SEC selects the PCAOB’s members.
B. The SEC must approve all PCAOB rules.
C. A violation of any PCAOB rule constitutes a violation of the 1934 Securities Exchange Act.
D. All of the above.

A

D. All of the above.

185
Q

According to the SEC, members of an issuer’s audit committee may not
A. Establish procedures for employees to anonymously report fraud.
B. Be responsible for the compensation of any registered public accounting firm employed by the registrant to provide an audit report.
C. Accept any consulting, advisory, or other compensatory fee from the registrant for services other than as a member of the board.
D. Engage independent counsel as deemed necessary to carry out their duties.

A

C. Accept any consulting, advisory, or other compensatory fee from the registrant for services other than as a member of the board.

All audit committee members must be independent of the corporation, meaning that fees such as these are impermissible.

186
Q

How many audits of public companies per year does a CPA firm that is registered with the Public Company Accounting Oversight Board (PCAOB) have to perform before it receives an annual inspection from the PCAOB?
A. One audit.
The correct answer is 100 audits. Firms performing fewer than 100 audits are to be inspected once every three years.
B. More than 10 audits.
C. More than 50 audits.
D. More than 100 audits.

A

D. More than 100 audits.

The PCAOB is required to inspect annually any registered CPA firm that audits more than 100 public companies.

187
Q

A person identified as an audit committee financial expert of an issuer generally must have acquired the attributes of a financial expert through any of the following experiences, except

A.  As a principal financial officer, principal accounting officer, controller, public accountant, or auditor.
B.  Serving on at least one other issuer's audit committee or disclosure committee of the board of directors.
C.  Actively supervising a principal financial officer or principal accounting officer.
D.  Assessing performance of the public accountants with respect to preparation, auditing, or evaluation of financial statements.
A

B. Serving on at least one other issuer’s audit committee or disclosure committee of the board of directors.

According to SEC rules promulgated pursuant to Section 407 of SOX, serving on two audit committees at once does not mean that the person is a financial expert. He or she might have been a professional bowler (only) before taking the two positions.

188
Q

Which of the following statements is correct regarding disclosure of client working papers prepared by a CPA?
A. Working papers may NOT be transferred to another accountant without the client’s permission.
B. Working papers may NOT be turned over to a CPA quality review team without the client’s permission.
C. Working papers may NOT be disclosed under a federal court subpoena without the client’s permission.
D. Working papers may NOT be disclosed to any third parties without the client’s permission.

A

A. Working papers may NOT be transferred to another accountant without the client’s permission.

Because of the confidentiality duty that a CPA owes a client, working papers may not be transferred to another accountant (typically a buyer of the accountant’s business) without the client’s permission.

189
Q

A government internal audit function is presumed to be free from organizational independence impairments for reporting internally when the head of the organization
A. Is not accountable to those charged with governance.
B. Performs auditing procedures that are consistent with generally accepted accounting principles.
C. Is a line-manager of the unit under audit.
D. Is removed from political pressure to conduct audits objectively, without fear of political reprisal.

A

D. Is removed from political pressure to conduct audits objectively, without fear of political reprisal.

190
Q

Sue’s firm was hired to audit a Reno County project that used federal grant money to attempt to create jobs for people on welfare. Sue was in charge of the audit, and her team found many questionable practices. When the chief administrator of Reno County’s government heard about Sue’s preliminary findings, he called her into his office and told her that her firm would lose every single audit contract it had with every single unit of Reno County government if he was not pleased with Sue’s audit report. This is an example of:
A. A potential personal impairment of independence.
B. A potential external impairment of independence.
C. A potential organizational impairment of independence.
D. None of the above.

A

B. A potential external impairment of independence.

The potential impairment threat is external, stemming from the threatened firing.

191
Q

The controller of a small utility company has interviewed audit firms proposing to perform the annual audit of their employee benefit plan. According to the guidelines of the Department of Labor (DOL), the selected auditor must be
A. The firm that proposes the lowest fee for the work required.
B. Independent for purposes of examining financial information required to be filed annually with the DOL.
C. Included on the list of firms approved by the DOL.
D. Independent of the utility company and NOT relying on its services.

A

B. Independent for purposes of examining financial information required to be filed annually with the DOL.

DOL guidelines do require independent for such work. Although DOL independence rules differ from AICPA rules, independence is still required.

192
Q

The Chicago office of the ABC accounting firm performs the financial statement audit of Ebben Co. Assuming that the IFAC Code applies to the relationship, who among the following must comply with the Code’s rules on independence?
A. Tip, a tax partner in ABC’s Miami office who does no work for Ebben Co.
B. Sally, a senior partner in ABC’s New York headquarters who has no supervisory authority over anyone in ABC’s Chicago office.
Sally appears to have no ability to directly influence the outcome of the engagement given her lack of direct involvement and lack of supervisory authority.
C. Ted, a retired auditor whom ABC hired to help its audit team because of his years of experience in auditing companies in Ebben Co’s line of business.
D. All of the above.

A

C. Ted, a retired auditor whom ABC hired to help its audit team because of his years of experience in auditing companies in Ebben Co’s line of business.

Ted is covered. An expert who functions as part of the engagement team must meet independence rules. It would be an obvious threat to compliance with fundamental principles if that were not the case.

193
Q
An audit client's CFO gave Tom, a member of the Tilden accounting firm's audit team, wonderful tickets to a championship soccer match in Europe. Later, when the CFO wished to press an aggressive earnings recognition position and Tom balked, Tom said: "Just because you gave me some soccer tickets doesn't mean I am going to give you everything you want." The CFO responded: "Maybe your state board of accountancy would be interested in knowing about those tickets." If the IFAC Code applied, which threats to compliance with fundamental principles would be present?
	A.  Self-interest threats.
	B.  Self-review threats.
	C.  Intimidation threats.
	D.  A and C.
A

D. A and C.

The gift coupled with the threat present BOTH a self-interest threat and an intimidation threat to Tom’s compliance with fundamental principles.

194
Q

While performing certain non-audit services for an insurance company, a professional accountant is asked to recommend the appropriate accounting treatment for a weather hedge transaction. The accountant has worked with financial hedges but has no experience with weather hedges. Which of the following actions by the accountant would be in compliance with the IFAC Code of Ethics for Professional Accountants?
A. Agree to recommend the appropriate accounting treatment after performing sufficient research on weather hedges.
B. Refuse to conduct the research and make a recommendation, because of insufficient experience.
C. Refuse to conduct the research and make a recommendation, because of a conflict of interest.
D. Agree with the accounting treatment recommended by the company’s hedge fund trader.

A

A. Agree to recommend the appropriate accounting treatment after performing sufficient research on weather hedges.

This is the best of these four options, especially since the others all involve making recommendations without having done any research. There is no independence issue here, given that the accountant is not doing any audit work.

195
Q

Mill Co. uses a batch processing method to process its sales transactions.
Data on Mill’s sales transaction tape are electronically sorted by customer number and are subjected to programmed edit checks in preparing its invoices, sales journals, and updated customer account balances.

One of the direct outputs of the creation of this tape most likely would be a

A.  Report showing exceptions and control totals.
B.  Printout of the updated inventory records.
C.  Report showing overdue accounts receivable.
D.  Printout of the sales price master file.
A

A. Report showing exceptions and control totals.

The results of a programmed edit check of sales transactions would include a report showing exceptions and control totals for each batch processed.

196
Q

In a computerized payroll system environment, an auditor would be least likely to use test data to test controls related to
A. Missing employee numbers.
B. Proper approval of overtime by supervisors.
C. Time tickets with invalid job numbers.
D. Agreement of hours per clock cards with hours on time tickets.

A

B. Proper approval of overtime by supervisors.

Test data involve the use of fictitious transactions to test whether application controls are functioning properly. They would be used to test controls related to missing employee numbers, time tickets with invalid job numbers, and agreement of hours per clock cards with hours on time tickets.

They would be LEAST likely to be used to test controls related to proper approval of overtime by supervisors, which are more likely to occur outside the application.

197
Q

Able Co. uses an online sales order processing system to process its sales transactions. Able’s sales data are electronically sorted and subjected to edit checks.
A direct output of the edit checks most likely would be a

A.  Report of all missing sales invoices.
B.  File of all rejected sales transactions.
C.  Printout of all user code numbers and passwords.
D.  List of all voided shipping documents.
A

B. File of all rejected sales transactions.

198
Q
If a control total were computed on each of the following data items, which would best be identified as a hash total for a payroll EDP application?
	A.  Total debits and total credits.
	B.  Net pay.
	C.  Department numbers.
	D.  Hours worked.
A

C. Department numbers.

A hash total is a meaningless total computed to verify the accuracy and completeness of input. Assigned department numbers, when totaled, would provide an example of a hash total.

199
Q
An auditor concluded that no excessive costs for idle plant were charged to inventory. This conclusion is most likely related to the auditor's goal of obtaining evidence about which assertion regarding inventory at the end of the period?
	A.  Valuation and allocation.
	B.  Completeness.
	C.  Existence.
	D.  Rights and obligations.
A

A. Valuation and allocation.

200
Q

To obtain assurance that all inventory items in a client’s inventory listing are valid, an auditor most likely would agree
A. Inventory tags noted during the auditor’s observation to items listed in receiving reports and vendors’ invoices.
B. Items listed in receiving reports and vendor’s invoices to the inventory listing.
C. Inventory tags noted during the auditor’s observation to items in the inventory listing.
D. Items in the inventory listing to inventory tags and the auditor’s recorded count sheets.

A

D. Items in the inventory listing to inventory tags and the auditor’s recorded count sheets.

201
Q

Periodic or cycle counts of selected inventory items are made at various times during the year rather than a single inventory count at year end.
Which of the following is necessary if the auditor plans to observe inventories at interim dates?

A.  Complete recounts by independent teams are performed.
B.  Perpetual inventory records are maintained.
C.  Unit cost records are integrated with production accounting records.
D.  Inventory balances are rarely at low levels.
A

B. Perpetual inventory records are maintained.

If inventory is to be observed at an interim date, perpetual inventory records must be maintained. Such records will provide the book balances against which the physical count can be compared.

202
Q
An auditor most likely would inspect loan agreements under which an entity's inventories are pledged in order to support management's financial statement assertion of
	A.  Existence or occurrence.
	B.  Completeness.
	C.  Presentation and disclosure.
	D.  Valuation or allocation.
A

C. Presentation and disclosure.

An auditor would inspect loan documents to verify that inventories have been pledged in support of the presentation and disclosure assertions.

203
Q
An auditor most likely would analyze inventory turnover rates to obtain evidence concerning management's assertions about
	A.  Existence or occurrence.
	B.  Rights and obligations.
	C.  Completeness.
	D.  Valuation or allocation.
A

D. Valuation or allocation.

Analysis of inventory turnover rates would provide evidence supporting valuation/allocation. A reduction in the turnover rate, for example, might indicate problems with obsolete inventory.
204
Q

Which of the following auditing procedures most likely would provide assurance about a manufacturing entity’s inventory valuation?
A. Testing the entity’s computation of standard overhead rates.
B. Obtaining confirmation of inventories pledged under loan agreements.
C. Reviewing shipping and receiving cut-off procedures for inventories.
D. Tracing test counts to the entity’s inventory listing.

A

A. Testing the entity’s computation of standard overhead rates.

Gaining assurance about the valuation of inventory requires, among other things, determining that inventories have been properly priced at the lower of cost or market. The determination of cost is obtained by reviewing current production costs, which might include testing the entity’s computation of standard overhead rates.

205
Q

While observing a client’s annual physical inventory, an auditor recorded test counts for several items and noticed that certain test counts were higher than the recorded quantities in the client’s perpetual records.
This situation could be the result of the client’s failure to record

A.  Purchase discounts.
B.  Purchase returns.
C.  Sales.
D.  Sales returns.
A

D. Sales returns.

Failure to record sales returns would result in actual inventory quantities (and, thus, test counts) being higher than recorded quantities in the client’s perpetual records.

206
Q

Which of the following procedures would be most appropriate for testing the completeness assertion as it applies to inventory?
A. Scanning perpetual inventory, production, and purchasing records.
B. Examining paid vendor’s invoices.
C. Tracing inventory items from the tag listing back to the physical inventory quantities.
D. Performing cut-off procedures for shipping and receiving.

A

D. Performing cut-off procedures for shipping and receiving.

The completeness assertion pertains to transactions that have NOT been recorded or are missing. Performing cut-off procedures for shipping and receiving enables the auditor to detect late transactions that may not have been recorded in the proper period and may be missing from the current (audit) year.

207
Q

A portion of a client’s inventory is in public warehouses. Evidence of the existence of this merchandise can most efficiently be acquired through which of the following methods?

A.  Observation.
B.  Confirmation.
C.  Calculation.
D.  Inspection.
A

B. Confirmation.

When inventory is held in a public warehouse, the auditor would ordinarily obtain direct confirmation from the custodian.

208
Q

The purpose of tracing a sample of inventory tags to a client’s computerized listing of inventory items is to determine whether the inventory items

A.  Represented by tags were included on the listing.
B.  Included on the listing were properly counted.
C.  Represented by tags were reduced to lower of cost or market.
D.  Included in the listing were properly valued.
A

A. Represented by tags were included on the listing.

The direction of the test is critical to the resulting inference. This test involves tracing from a type of source document (the count “tags”) to the accounting records (since the computerized listing of inventory items is, in effect, the subsidiary ledger that supports the inventory-adjusted general ledger balance). Hence, this test addresses the “completeness” assertion by dealing with the risk of omission.

209
Q
In building an electronic data interchange (EDI) system, what process is used to determine which elements in the entity's computer system correspond to the standard data elements?
	A.  Mapping.
	B.  Translation.
	C.  Encryption.
	D.  Decoding.
A

A. Mapping.

In an EDI system, a standard format is adopted. Mapping is the process by which the elements in the client’s computer system are related to the standard data elements.

210
Q
An auditor would least likely use computer software to
	A.  Access client data files.
	B.  Prepare spreadsheets.
	C.  Assess EDP control risk.
	D.  Construct parallel simulations.
A

C. Assess EDP control risk.

Computer software would not be used to assess EDP control risk. Assessments of control risk are a matter of auditor judgment; the use of computer software will not facilitate that judgment.

211
Q

Which of the following is a general control that would most likely assist an entity whose systems analyst left the entity in the middle of a major project?
A. Grandfather-father-son record retention.
B. Input and output validation routines.
C. Systems documentation.
D. Check digit verification.

A

C. Systems documentation.

Systems documentation is a general control that would assist an entity whose systems analyst left in the middle of a major project. Such documentation would be prepared for each application system and would include narratives and flowcharts. It would document the work completed to date on the project (ideally) and enable an analyst to take over.

212
Q

Which of the following most likely represents a significant deficiency in the internal control structure?
A. The systems analyst reviews applications of data processing and maintains systems documentation.
B. The systems programmer designs systems for computerized applications and maintains output controls.
C. The control clerk establishes control over data received by the EDP department and reconciles control totals after processing.
D. The accounts payable clerk prepares data for computer processing and enters the data into the computer.

A

B. The systems programmer designs systems for computerized applications and maintains output controls.

In a well-designed system of internal control, the following duties must be segregated: systems analysis, programming, computer operations, transaction authorization, library functions, and data control.

213
Q
Which of the following is not a major reason for maintaining an audit trail for a computer system?
	A.  Deterrent to irregularities.
	B.  Monitoring purposes.
	C.  Analytical procedures.
	D.  Query answering.
A

C. Analytical procedures.

Maintaining an audit trail for a computer system provides a deterrent to irregularities, facilitates monitoring, and enables queries to be answered. It does NOT enable analytical procedures to be performed.

214
Q

n auditor most likely would test for the presence of unauthorized EDP program changes by running a
A. Program with test data.
B. Check digit verification program.
C. Source code comparison program.
D. Program that computes control totals.

A

C. Source code comparison program.

A comparison of the compiled object program code with the original program would reveal unauthorized program changes.

215
Q

Which of the following controls most likely would assure that an entity can reconstruct its financial records?
A. Hardware controls are built into the computer by the computer manufacturer.
B. Backup diskettes or tapes of files are stored away from originals.
C. Personnel who are independent of data input perform parallel simulations.
D. System flowcharts provide accurate descriptions of input and output operations.

A

B. Backup diskettes or tapes of files are stored away from originals.

Storage of backup diskettes, tapes, or disk files on remote computers away from originals will provide assurance that an entity can reconstruct its financial records.

216
Q

When evaluating internal control of an entity that processes sales transactions on the Internet, an auditor would be most concerned about the
A. Lack of sales invoice documents as an audit trail.
B. Potential for computer disruptions in recording sales.
C. Inability to establish an integrated test facility.
D. Frequency of archiving and data retention.

A

B. Potential for computer disruptions in recording sales.

217
Q

Which of the following is a password security problem?
A. Users are assigned passwords when accounts are created, but do not change them.
B. Users have accounts on several systems with different passwords.
C. Users copy their passwords onto notepaper, which is kept in their wallets.
D. Users select passwords that are not listed in any online dictionary.

A

A. Users are assigned passwords when accounts are created, but do not change them.

218
Q
Which of the following passwords would be most difficult to crack?
	A.  OrCa!FlSi
	B.  language
	C.  12 HOU.S.E 24
	D.  pass56word
A

A. OrCa!FlSi

Guidelines for choosing a “secure” password include the following:
the password should be at least seven characters in length;
the password should include special characters, such as punctuation marks or symbols;
the password should be a mixture of uppercase and lowercase letters;
the password should be unique.

219
Q

Which of the following statements is correct concerning internal control in an electronic data interchange (EDI) system?
A. Preventive controls are generally more important than detective controls in EDI systems.
B. Control objectives for EDI systems are generally different from the objectives for other information systems.
C. Internal controls in EDI systems rarely permit control risk to be assessed at below the maximum.
D. Internal controls related to the segregation of duties are generally the most important controls in EDI systems.

A

A. Preventive controls are generally more important than detective controls in EDI systems.

220
Q

Which of the following statements is correct concerning the security of messages in an electronic data interchange (EDI) system?
A. When the confidentiality of data is the primary risk, message authentication is the preferred control, rather than encryption.
B. Encryption performed by physically secure hardware devices is more secure than encryption performed by software.
C. Message authentication in EDI systems performs the same function as segregation of duties in other information systems.
D. Security at the transaction phase in EDI systems is not necessary because problems at that level will usually be identified by the service provider.

A

B. Encryption performed by physically secure hardware devices is more secure than encryption performed by software.

221
Q

Which of the following is usually a benefit of transmitting transactions in an electronic data interchange (EDI) environment?
A. A compressed business cycle with lower year-end receivables balances.
B. A reduced need to test computer controls related to sales and collections transactions.
C. An increased opportunity to apply statistical sampling techniques to account balances.
D. No need to rely on third-party service providers to ensure security.

A

A compressed business cycle with lower year-end receivables balances.

An electronic data interchange environment enables the business cycle to be reduced (or compressed). For example, sales may be invoiced immediately, with the resultant speed-up of cash collections and reduction of receivable balances.

222
Q
Which of the following computer-assisted auditing techniques processes client input data on a controlled program under the auditor's control to test controls in the computer system?
	A.  Test data.
	B.  Review of program logic.
	C.  Integrated test facility.
	D.  Parallel simulation.
A

D. Parallel simulation.

223
Q

Which of the following is usually a benefit of using electronic funds transfer for international cash transactions?
A. Improvement of the audit trail for cash receipts and disbursements.
B. Creation of self-monitoring access controls.
C. Reduction of the frequency of data entry errors.
D. Off-site storage of source documents for cash transactions.

A

C. Reduction of the frequency of data entry errors.

224
Q

Which of the following statements most likely represents a disadvantage for an entity that keeps microcomputer-prepared data files rather than manually prepared files?
A. Random error associated with processing similar transactions in different ways is usually greater.
B. It is usually more difficult to compare recorded accountability with physical count of assets.
C. Attention is focused on the accuracy of the programming process, rather than on errors in individual transactions.
D. It is usually easier for unauthorized persons to access and alter the files.

A

transactions.

D. It is usually easier for unauthorized persons to access and alter the files.

225
Q

Which of the following is an engagement attribute for an audit of an entity that processes most of its financial data in electronic form, without any paper documentation?
A. Discrete phases of planning, interim, and year-end field work.
B. Increased effort to search for evidence of management fraud.
C. Performance of audit tests on a continuous basis.
D. Increased emphasis on the completeness assertion

A

C. Performance of audit tests on a continuous basis.

An audit of an entity with primarily electronic data systems would be more likely to include continuous audit testing because in such environments transactions or accounting records may be available on a temporary basis and in machine-readable form only. As a result, testing would need to be performed continuously, rather than at one time.

226
Q

Processing data through the use of simulated files provides an auditor with information about the operating effectiveness of control policies and procedures.
One of the techniques involved in this approach makes use of

A.  Input validation.
B.  Program code checking.
C.  Controlled re-processing.
D.  Integrated test facility.
A

D. Integrated test facility.

227
Q
An auditor using audit software probably would be least interested in which of the following fields in a computerized perpetual inventory file?
	A.  Economic order quantity.
	B.  Warehouse location.
	C.  Date of last purchase.
	D.  Quantity sold.
A

A. Economic order quantity.

The auditor’s primary objective in reviewing the perpetual inventory file is to verify the financial statement assertions pertaining to inventory. As a result, the auditor would focus on such fields as warehouse location, date of last purchase, and quantity sold. The auditor would be LEAST interested in economic order quantity, which is not relevant to verifying the year-end inventory.

228
Q

Which of the following computer-assisted auditing techniques allows fictitious and real transactions to be processed together without client operating personnel being aware of the testing process?
A. Integrated test facility.
B. Input controls matrix.
C. Parallel simulation.
Fictitious and real transactions are processed together without the knowledge of operating personnel in an integrated test facility. In a parallel simulation, the auditor writes a program and processes client data. The result is then compared to the company’s and differences are investigated.
D. Data entry monitor.

A

A. Integrated test facility.

Fictitious and real transactions are processed together without the knowledge of operating personnel in an integrated test facility.

229
Q

Which of the following is the primary reason that many auditors hesitate to use embedded audit modules?
A. Embedded audit modules cannot be protected from computer viruses.
B. Auditors are required to monitor embedded audit modules continuously to obtain valid results.
C. Embedded audit modules can easily be modified through management tampering.
D. Auditors are required to be involved in the system design of the application to be monitored.

A

D. Auditors are required to be involved in the system design of the application to be monitored.

230
Q

In auditing an entity’s computerized payroll transactions, an auditor would be least likely to use test data to test controls concerning

A.  Overpayment of employees for hours not worked.
B.  Control and distribution of unclaimed checks.
    C.  Withholding of taxes and Social Security contributions.
D.  Missing employee identification numbers.
A

B. Control and distribution of unclaimed checks.

“Test data” would not be helpful in evaluating physical security controls over unclaimed checks or other documents.

231
Q

When an auditor tests the internal controls of a computerized accounting system, which of the following is true of the test data approach?
A. Test data are coded to a dummy subsidiary so they can be extracted from the system under actual operating conditions.
B. Test data programs need not be tailor-made by the auditor for each client’s computer applications.
C. Test data programs usually consist of all possible valid and invalid conditions regarding compliance with internal controls.
D. Test data are processed with the client’s computer and the

A

A. Test data are coded to a dummy subsidiary so they can be extracted from the system under actual operating conditions.

232
Q

Which of the following procedures does a CPA normally perform first in a review engagement in accordance with Statements on Standards for Accounting and Review Services (SSARS)?
A. Inquiry regarding the client’s principles and practices and the method of applying them.
B. Inquiry concerning the effectiveness of the client’s system of internal control.
C. Inquiry to identify transactions between related parties and management.
D. Inquiry of the client’s professional advisors, including bankers, insurance agents, and consultants.

A

A. Inquiry regarding the client’s principles and practices and the method of applying them.

233
Q

Smith, CPA, has been asked to issue a review report on the balance sheet of Cone Company, a nonpublic entity, and not on the other related financial statements.
Smith may do so only if

A.  Smith compiles and reports on the related statements of income, retained earnings, and cash flows.
B.  Smith is not aware of any material modifications needed for the balance sheet to conform with GAAP.
C.  The scope of Smith's inquiry and analytical procedures is not restricted.
D.  Cone is a new client and Smith accepts the engagement after the end of Cone's fiscal year.
A

C. The scope of Smith’s inquiry and analytical procedures is not restricted.

A review engagement, like an audit, may be performed on a single financial statement as long as the scope of the accountant’s procedures is not restricted. In the case of a review, the scope of the inquiry and analytical procedures must not be restricted.

234
Q

An accountant should perform analytical procedures during an engagement to
Compile a nonpublic entity’s financial statements Review a nonpublic entity’s financial statements
No Yes
Yes Yes
Yes No
No No

A

Compile a nonpublic entity’s financial statements No Review a nonpublic entity’s financial statements Yes

The accountant is not required to perform analytical procedures or make inquiries in a compilation. The accountant IS required to perform both analytical procedures and inquiries in a review.

235
Q
May an accountant accept an engagement to compile or review the financial statements of a not-for-profit entity if the accountant is unfamiliar with the specialized industry accounting principles, but plans to obtain the required level of knowledge before compiling or reviewing the financial statements?
  Compilation  	  Review  
	 No 	 No 
	 Yes 	 No 
	 No 	 Yes 
	 Yes 	 Yes
A

Compilation Yes Review Yes

236
Q

If management refuses to correct the financial statement presentations, the accountant should

A.  Issue an adverse opinion. An adverse opinion is associated with an audit, not a review.
B.  Issue an "except for" qualified opinion.
C.  Disclose this departure from generally accepted accounting principles in a separate paragraph of the report.
D.  Express only limited assurance on the financial statement presentations.
A

C. Disclose this departure from generally accepted accounting principles in a separate paragraph of the report.

If the accountant becomes aware of a GAAP departure during a review engagement, the review report should disclose this departure in a separate paragraph of the report.

237
Q

Baker, CPA, was engaged to review the GAAP-based financial statements of Hall Company, a nonpublic entity. Evidence came to Baker’s attention that indicated substantial doubt as to Hall’s ability to continue as a going concern.
The principal conditions and events that caused the substantial doubt have been fully disclosed in the notes to Hall’s financial statements.
Which of the following statements best describes Baker’s reporting responsibility concerning this matter?

A.  Baker is not required to modify the accountant's review report.
B.  Baker is not permitted to modify the accountant's review report.
C.  Baker should issue an accountant's compilation report instead of a review report.
D.  Baker should express a qualified opinion in the accountant's review report.
A

A. Baker is not required to modify the accountant’s review report.

238
Q

An accountant who had begun an audit of the financial statements of a nonpublic entity was asked to change the engagement to a review because of a restriction on the scope of the audit. If there is reasonable justification for the change, the accountant’s review report should include reference to the
Scope limitation that caused the changed engagement Original engagement that was agreed to
Yes No
No Yes
No No
Yes Yes

A

Scope limitation that caused the changed engagement No

Original engagement that was agreed to No

239
Q

Baker, CPA, was engaged to review the financial statements of Hall Co., a nonpublic entity. During the engagement Baker uncovered a complex scheme involving client illegal acts and fraud that materially affect Hall’s financial statements.
If Baker believes that modification of the standard review report is not adequate to indicate the deficiencies in the financial statements, Baker should

A.  Disclaim an opinion.
B.  Issue an adverse opinion.
C.  Withdraw from the engagement.
D.  Issue a qualified opinion.
A

C. Withdraw from the engagement.

If Baker believes that modification of the review report is not adequate to indicate the deficiencies in the financial statements, Baker should withdraw from the engagement.

240
Q

Which of the following situations would preclude an accountant from issuing a review report on a company’s financial statements in accordance with Statements on Standards for Accounting and Review Services (SSARS)?
A. The owner of a company is the accountant’s father.
B. The accountant was engaged to review only the balance sheet.
C. Land has been recorded at appraisal value instead of historical cost.
D. Finished-goods inventory does not include any overhead amounts.

A

A. The owner of a company is the accountant’s father.

An accountant must be independent to perform a review. An accountant is not likely to be independent if the client’s owner is his/her father.

241
Q

Which of the following procedures most likely would not be included in a review engagement of a nonpublic entity?
A. Obtaining a management representation letter.
B. Considering whether the financial statements conform with GAAP.
C. Assessing control risk.
D. Inquiring about subsequent events.

A

C. Assessing control risk.

In a review engagement, the accountant would not assess control risk. Assessment of control risk is performed in an audit engagement.

242
Q

Financial statements of a nonpublic entity that have been reviewed by an accountant should be accompanied by a report stating that
A. The scope of the inquiry and analytical procedures performed by the accountant has not been restricted.
B. All information included in the financial statements is the representation of the management of the entity.
C. A review includes examining, on a test basis, evidence supporting the amounts and disclosures in the financial statements.
D. A review is greater in scope than a compilation, the objective of which is to present financial statements that are free of material misstatements.

A

B. All information included in the financial statements is the representation of the management of the entity.

A review includes a statement that management is responsible for the preparation and fair presentation of the financial statements in accordance with the applicable financial reporting framework.

243
Q

Performing inquiry and analytical procedures is the primary basis for an accountant to issue a
A. Report on compliance with requirements governing major federal assistance programs in accordance with the Single Audit Act.
B. Review report on prospective financial statements that present an entity’s expected financial position, given one or more hypothetical assumptions.
C. Management advisory report prepared at the request of a client’s audit committee.
D. Review report on comparative financial statements for a nonpublic entity in its second year of operations.

A

D. Review report on comparative financial statements for a nonpublic entity in its second year of operations.

244
Q

Before issuing a report on the compilation of financial statements of a nonpublic entity, the accountant should
A. Apply analytical procedures to selected financial data to discover any material misstatements.
B. Corroborate at least a sample of the assertions management has embodied in the financial statements.
C. Inquire of the client’s personnel whether the financial statements omit substantially all disclosures.
D. Read the financial statements to consider whether the financial statements are free from obvious material errors.

A

D. Read the financial statements to consider whether the financial statements are free from obvious material errors.

245
Q

An accountant is required to comply with the provisions of Statements on Standards for Accounting and Review Services (SSARS) when
When engaged to compile financial statements generated through the use of computer software Reproducing client-prepared financial statements, without modification, for the client
Yes Yes
Yes No
No Yes
No No

A

Yes No

The accountant must comply with the SSARSs when engaged to compile financial statements, even if they are generated through the use of computer software, because the accountant’s expertise was necessary to create the financial statements. (Even if the accountant is engaged to “prepare” financial statements, rather than “compile” them, the SSARSs still are applicable.) The accountant is not required to follow the SSARS if simply retyping already prepared financial statements. This is not considered to be a compilation (or preparation) engagement.

246
Q

An accountant compiled the financial statements of a nonissuer in accordance with Statements on Standards for Accounting and Review Services (SSARS). If the accountant has an ownership interest in the entity, which of the following statements is correct?
A. The accountant should refuse the compilation engagement.
B. A report need not be issued for a compilation of a nonissuer.
C. The accountant is required to include the disclaimer “I am an owner of the entity” in the report.
D. The accountant is required to include the statement “I am not independent with respect to the entity” in the compilation report.

A

D. The accountant is required to include the statement “I am not independent with respect to the entity” in the compilation report.

247
Q

An accountant compiles the financial statements of a nonissuer and issues the standard compilation report. Although not specifically stated in this report, it is implied that
A. The accountant has not audited or reviewed the financial statements.
B. Substantially all disclosures required by GAAP are included in the financial statements.
C. The financial statements should not be used to obtain credit.
D. Management is responsible for the financial statements that are the object of the compilation engagement.

A

B. Substantially all disclosures required by GAAP are included in the financial statements.

When the financial statements omit substantially all disclosures required by GAAP, the accountant’s compilation report should state that fact. If the compilation report does not include a statement regarding the omission of such disclosures, then it is implied that the financial statements include the appropriate disclosures.

248
Q

When compiling the financial statements of a nonpublic entity, an accountant should
A. Review agreements with financial institutions for restrictions on cash balances.
In a compilation, an accountant is not required to review agreements with financial institutions for restrictions on cash balances. This is a substantive procedure utilized in an audit.
B. Understand the accounting principles and practices of the entity’s industry.
C. Inquire of key personnel concerning related parties and subsequent events.
D. Perform ratio analyses of the financial data of comparable prior periods.

A

B. Understand the accounting principles and practices of the entity’s industry.

In a compilation, an accountant should possess an understanding of the accounting principles and practices of the industry in which the entity operates.

249
Q

An accountant has been engaged to compile the financial statements of a nonpublic entity in accordance with Statements on Standards for Accounting and Review Services (SSARS).
Do the SSARSs require that the compilation report be printed on the accountant’s letterhead and that the report be manually signed by the accountant?

  Printed on the accountant's letterhead  	  Manually signed by the accountant  
	 Yes 	 Yes 
	 Yes 	 No 
	 No 	 Yes 
	 No 	 No
A

Printed on the accountant’s letterhead No

Manually signed by the accountant No

250
Q

Jones Retailing, a nonpublic entity, has asked Winters, CPA, to compile financial statements that omit substantially all disclosures required by generally accepted accounting principles. Winters may compile such financial statements provided the
A. Reason for omitting the disclosures is explained in the engagement letter and acknowledged in the management representation letter.
B. Financial statements are prepared on a comprehensive basis of accounting other than generally accepted accounting principles.
C. Distribution of the financial statements is restricted to internal use only.
D. Omission is not undertaken to mislead the users of the financial statements and is properly disclosed in the accountant’s report.

A

D. Omission is not undertaken to mislead the users of the financial statements and is properly disclosed in the accountant’s report

251
Q

Compiled financial statements should be accompanied by a report stating that
A. A compilation is substantially less in scope than a review or an audit in accordance with generally accepted auditing standards.
A compilation report does not include a statement that a compilation is substantially less in scope than a review or an audit, nor does it include any reference to GAAS. A compilation report does not provide any assurance.
B. The accountant does not express an opinion but expresses only limited assurance on the compiled financial statements.
C. The accountant did not perform any procedures to verify the accuracy or completeness of the information provided by management.
D. The accountant has compiled the financial statements in accordance with standards established by the Auditing Standards Board.

A

C. The accountant did not perform any procedures to verify the accuracy or completeness of the information provided by management.

A compilation report should include the following statement: “I (We) did not audit or review the financial statements nor was (were) I (we) required to perform any procedures to verify the accuracy or completeness of the information provided by management.”

252
Q

Miller, CPA, is engaged to compile the financial statements of Web Co., a nonpublic entity, in conformity with the income tax basis of accounting.
If Web’s financial statements do not disclose the basis of accounting used, Miller should
A. Disclose the basis of accounting in the accountant’s compilation report.
B. Clearly label each page “Distribution Restricted – Material Modifications Required.”
C. Issue a special report describing the effect of the incomplete presentation.
D. Withdraw from the engagement and provide no further services to Web.

A

A. Disclose the basis of accounting in the accountant’s compilation report.

If the financial statements are prepared using a special purpose framework (such as the income tax basis) and the financial statements do not disclose this basis, the accountant’s compilation report should identify the basis used.

253
Q

When an accountant is engaged to compile a nonpublic entity’s financial statements that omit substantially all disclosures required by GAAP, the accountant should indicate in the compilation report that the financial statements are
A. Not designed for those who are uninformed about the omitted disclosures.
B. Prepared in conformity with a comprehensive basis of accounting other than GAAP.
C. Not compiled in accordance with Statements on Standards for Accounting and Review Services.
D. Special-purpose financial statements that are not comparable to those of prior periods.

A

A. Not designed for those who are uninformed about the omitted disclosures.

254
Q

Clark, CPA, compiled and properly reported on the financial statements of Green Co., a nonpublic entity, for the year ended March 31, 20X1. These financial statements omitted substantially all disclosures required by generally accepted accounting principles (GAAP).
Green asked Clark to compile the statements for the year ended March 31, 20X2, and to include all GAAP disclosures for the 20X2 statements only, but otherwise present both years’ financial statements in comparative form. What is Clark’s responsibility concerning the proposed engagement?

A.  Clark may not report on the comparative financial statements because the 20X1 statements are not comparable to the 20X2 statements that include the GAAP disclosures.
B.  Clark may report on the comparative financial statements provided the 20X1 statements do not contain any obvious material misstatements.
C.  Clark may report on the comparative financial statements provided an explanatory paragraph is added to Clark's report on the comparative financial statements.
D.  Clark may report on the comparative financial statements provided Clark updates the report on the 20X1 statements that do not include the GAAP disclosures.
A

A. Clark may not report on the comparative financial statements because the 20X1 statements are not comparable to the 20X2 statements that include the GAAP disclosures.

A CPA may not report on comparative financial statements when financial statements that both omit substantially all disclosures and include necessary disclosures are included. Financial statements that omit substantially all disclosures are not comparable to financial statements that include such disclosures.

255
Q

How does an accountant make the following representations when issuing the standard report for the compilation of a nonpublic entity’s financial statements?
The financial statements have not been audited The accountant has performed a compilation engagement on the financial statements
Implicitly Implicitly
Explicitly Explicitly
Implicitly Explicitly
Explicitly Implicitly

A

Explicitly Explicitly

The compilation report explicitly states that the financial statements have not been audited (or reviewed) and that the accountant has performed a compilation engagement.